Download as pdf or txt
Download as pdf or txt
You are on page 1of 151

MATHEMATICS-I

WWW.IMTSINSTITUTE.COM
IMTS
(ISO 9001-2008 Internationally Certified)

MATHEMATICS-I

MATHEMATICS-I

IMTSINSTITUTE.COM
MATHEMATICS-I

CONTENTS:

UNIT – 1: 01-42

Characteristic equation – Eigen Values and Eigen Vectors –


Properities – Problems – Rank of Matrix – Problems – Solutions of
simultaneous equations using matrices – consistency condition. Polynomial
equations – relation between roots and coefficients imaginary roots and
irrational roots – solving equations under given conditions – transformation
of equations.

UNIT – II: 43-68

Definition of a derivative , different types of differentiation – standard


formulae – successive differentiation – nth derivative – Leibnitz formula –
problems. Partial differentiation – Euler’s theorem – Curvature – Radius of
curvature in Cartesian co-ordinates.

UNIT - III : 69-99


  
2 2 2 a a

 sin  cos  tan x e x e


n n n n ax n x
Integration by Parts : xdx xdx , xdx , dx , dx
0 0 0 0 0

Definite integrals – properties – reduction formulae – Problems. Second


order differential equations with constant coefficients – Particular Integrals of
type e axV - where V is x or x 2 or cos ax or sin ax

IMTSINSTITUTE.COM
UNIT – IV : 100-117
Definition Complete, - Singular and General integrals solutions of
Standard types f (p, q) = 0, f ( x, p, q) = 0, f ( y, p, q) = 0, f ( z, p, q ) = 0, f 1

(x, p )= f 2 ( x, p) – Clariant’s form – Lagrange’s equation Pp + Qq = R –


Problems.

UNIT - V: 118-142

Definition – Laplace transform of standard function simple theorems –


problems inverse Laplace transform – Fourier coefficients – periodic
functions with period 2 p – half range series – cosine series – sine series –
problems.

UNIT QUESTIONS: 143-147

IMTSINSTITUTE.COM
MATHEMATICS-I 1

UNIT-I

1.0 Introduction
1.1 Objectives
1.2 Characteristic Equation
1.2.1 Eigen values and Eigen vectors
1.2.2 Characteristic roots
1.2.4 Cayley – Hamilton’s Theorem
1.2.5 Properties of Characteristics roots
1.2.6 Problems
1.3 Rank of a Matrix
1.3.1 Problems
1.4 Simultaneous equations using matrices
1.4.1 Elementary Transformation
1.4.2 Equivalent matrices
1.5 Relation between the roots and coefficients of an Equation

1.6 Imaginary and irrational roots

1.7 Transformation of equation

1.0 Introduction

The Source material of Mathematics is considered to be a source of study


organized according to a set plan and a learning guide rather than a source book of
information. The content is given as simple, brief, exact, accessible and definite. The
source of mathematics is satifying the demand of the examinations, especially new type
tests in mathematics. This syllabus is more suitable for the application of mathematics in
the field of technology. It is completely deal with applied oriented mathematics with
equation, derivatives, radius of curvature in Cartesian co-ordinates. Integration,
Singularities and Laplace transforms and Fourier series and all problems presented in a
very simple language with suitable examples.

IMTSINSTITUTE.COM
MATHEMATICS-I 2

1.1 Objectives

 To develop certain mathematical abilities and skills among students.

 To develop the knowledge of given topics which covered by syllabus.

 The pupils may have proper view of the subject covered all the syllabus mater to be
studied.

 To crease new knowledge to solve the problem given in the exercise by


understanding the solved examples.

 It helps the pupils to acquire the required information with speed and accuracy.

 To develop the skills of measuring, thinking, analyzing logically.

 To crease the knowledge of application oriented mathematics and solving problems


in allied mathematics related in all other service subjects.

1.2 Characteristic equation of a matrix

Let A be a n × n square matrix over a field F and I be the unit matrix of the
same order. Let I be an unknown. Then determinant A – I  is called the
characteristic Polynomial of a matrix A. The equation det.A – I  = 0 is called the
characteristic equation of the matrix A. The roots of this equation are called the
characteristic roots of the matrix A. Characteristic roots are also called latent roots or
eigen values.

The characteristic vectors of a Matrix

Let A be a n × n matrix. Let X be any non-zero column vector,

x1
.
.
i.e X = .
.
xn
then any solution of the equation AX = X other than X = 0 corresponding to some
particular value of  is called a characteristic vector or Eigen vector or latent vector.

The equation AX = I X may be written as ( A = I) X = 0. This equation gives a set


of linear homogenous equations. If it is to possess a non-trivial solution, then.A – I = 0.
Then  is a characteristic root of the matrix A. Hence to find all the characteristic vectors,
we adopt the following procedure
.
Step I :
Solve A – I = 0 for 
Let 1, 2, 3 …………..n be the characteristic root of A.

Step II :
Find the non-trivial solution of A – I = 0 . This solution is the characteristic
vector corresponding to be characteristic value or characteristic roots of 

Step III :
Similarly find the characteristic vector corresponding to the characteristic roots.

IMTSINSTITUTE.COM
MATHEMATICS-I 3

Step IV :

Note that the number of linearly independent solution of (A – I ) X = 0 is equal to


n-Rank of A – I .

Problems:

Example : 1
1 2
1. Find the characteristic equation of A =
2 1
Solution:

The characteristic equation is A – I  = 0

1 2 1 0
(i.e) - = 0
2 1 0 1

1- 2
(i.e) = 0
2 1-

(i.e) (1- ) ( 1- ) – 4 = 0

 –2-3=0
2

Example : 2

2 0 1
Find the characteristic equation of A = 0 6 0
4 0 2

Solution:

The characteristic equation is A – I  = 0

2 0 4 1 0 0
(i.e) 0 6 0 -  0 1 0 = 0
4 0 2 0 0 1

2 0 4  0 0
(i.e) 0 6 0 - 0  0 = 0
4 0 2 0 0 

2- 0 4
(i.e) 0 6- 0 = 0
4 0 2-

= ( 2-  ) [ (6-) (2 -  ) – 0 ] – 0 + 4 [ 0 – 4 ( 6- ) ] = 0

IMTSINSTITUTE.COM
MATHEMATICS-I 4

( 2-  ) [ 12- 6 - 2  +  ) + 4 [ -24 + 4  ] = 0
2
=

( 2-  ) [ 12- 8 +  ) + 96 + 16  = 0
2
=

24 - 16  + 2  – 12  + 8  -  - 96 + 16  = 0
2 2 3
=

 - 10  + 12  + 72 = 0
3 2

1.2.3 Characteristic Roots:

0 1 1
Find the eigen value of A = - 4 4 2
4 - 3 -1

Solution:

The characteristic equation is A – I  = 0

0 1 1 1 0 0
(i.e) -4 4 2 -  0 1 0 = 0
4 -3 - 1 0 0 1

- 1 1
(i.e) -4 4- 2 = 0
4 - 3 -1- 

-  [ (4-) ( -1 -  0 + 6 ] – 1 [ -4 (-1- ) – 8 ] + 1[ 12-4(4-)] =0

-  ( 4 - 4 +  +  + 6 ) - ( - 4 + 4  – 8 ) + 1 ( 12- 16 + 4 ) = 0
2

-  +3 -2 =0
3 2

 -3 +2 =0
3 2

 (  - 3  +2 ) = 0
2

 = 0 and (  - 3  +2 ) = 0
2

 = 0 and (-1) ( -2)=0

 = 0,  = 1,  = 2

SELF – ASSESSMENT QUESTIONS : I

1. Find the eigen values and eigen vectors of the matrices

1 2 3 
a)
0 4 2 
 
0 0 7 

……………………………………………………………………………………..

IMTSINSTITUTE.COM
MATHEMATICS-I 5

……………………………………………………………………………………..

……………………………………………………………………………………..

Ans: 1, -4, 7 ; [k,0,0]; k [ 1, ½ 0 ]

1.2.4 Cayley Hamilton’s Theorem

Theorem :

Every square matrix satisfies its own characteristic equation


(i.e. If the characteristic polynomials is
 () =  + P1  + P2  + … + P n-1  + Pn
n n-1 n-2

then,  (A) = 0 
n n-1 n-2
A + P1 A + P2 A + … + P n-1 A + P n I = 0
Proof:

 () = | A - I |

a11 -  a12 … a1n


a21 a22 -  … a2n
a31 a32 A33 -  … a3n
. . . . .
. . . . .
. . . . .
an1 an2 an3 … ann - 

= P0  + P1  + P2 
n n-1 n-2
- … + P n ( Say)

Then we have to prove that

 () = P0 A + P1 A
n n-1 n-2
+ P2 A + … + Pn I = 0

Consider ( A - I )* which is the adjacent of A -  I. Let us call it B. The elements of B are


minors of order (n-1) of A -  I. The element of B are of degree at most n-1 in  and hence
can be written as

B0  + B1 
n-1 n-2
+ …. + B n-1

Where B0, B1, B2, …. B n-1 are matrices of order n whose elements are polynomials
in the elements of A

We know that
(A-I) Adj. (A-I) = | A- I| I
=  () I

(A-I) (B0  + B1 
n-1 n-2
+ …. + B n-1 ) I
= ( P0  + P1  + P2  - … + P n ) I
n n-1 n-2

This is an identity in  . Therefore equating the coefficient of like powers of , we obtain

 B0 = P0 I
AB0 – B1 = P1 I
AB1 – B2 = P2 I
.
.
.

IMTSINSTITUTE.COM
MATHEMATICS-I 6

ABr-1 – Br = Pr I
.
.
.
Abn-1 – B = Pn I
n n-1
By pre-multiplying these equations with A , A … A, I respectively
n n-1 n-2
0 = P0 A + P1 A + P2 A -…+Pn I

Which prove Cayley – Hamilton’s Theorem

Note : : Cayley-Hamilton’s Theorem can be used to find the inverse of a Matrix.

By Cayley- Hamilton’s theorem,


n n-1 n-2
0 = P0 A + P1 A + P2 A - … + Pn I
-1
Multiplying both side by A , we have
n-1 n-2 n-2 -1
0= P0 A + P1 A + P2 A +… + Pn-2 + Pn A
1
-1 n-1 n-2 n-2 -1
A = ---- P0 A + P1 A + P2 A +… + Pn-2 + Pn A
n
P

Note : Cayley – Hamilton’s theorem can also be used in computation of arbitrary


integral power of A.

Example : 1

0 1 2 

Determine the characteristic roots of the matrix 1 0 1

 
 2 1 0 

0 1 2
Let A = 1 0 -1
2 -1 0

0 1 2 1 0 0
A - I = 1 0 -1 -  0 1 0
2 -1 0 0 0 1

- 1 2
= 1 - -1
2 -1 -

The characteristic equation of A is | A -  I | = 0

- 1 2
1 - -1 = 0
2 -1 -

  ( – 1 ) – 1 ( -  + 2 ) + 2 ( - 1 + 2  ) = 0
2

 +  +  - 2 -2 + 4  = 0
3

IMTSINSTITUTE.COM
MATHEMATICS-I 7

 –6+4=0
3

 ( – 4 ) – 2 (  - 2 ) = 0
2

( – 2 ) [  (  + 2 ) – 2 ] = 0

( – 2 ) (  + 2 – 2 ) = 0
2

 = 2, -2±4+8
2
= 2, - 2 ± 23
2
= 2, - 1 ± 3

 The characteristic roots are 2, - 1 ± 3

Example : 2

a h g
Find the eigen values of 0 b 0
0 0 c

a h g
Let A = 0 b 0
0 0 c

The characteristic equation is | A -  I | = 0

a- h g
(i.e ) = 0 b- 0
0 0 c- 
i.e. (a - ) (b - ) (c - ) = 0
 = a, b, c
the eigen values are a, b, c

Example : 3

Prove that the matrices A, B, C given below have the same characteristic values.

0 a b 0 b a 0 c c
A= a 0 c B= b 0 c C= c 0 a
b c 0 a c 0 b c 0

Solution :
The characteristic equation of A is A -  I = 0

- a b
|A-I|=0  a - c = 0
B c -

 -  ( – c ) – a ( - a - bc ) + b (ac + b ) = 0
2 2

 -  + c + a  + abc + abc + b  = 0
3 2 2 2

  -  ( a + b + c ) – 2 abc = 0
3 2 2 2

The characteristic equation of B is | B - I | = 0

- b c
|B-I|=0 ; 0 - c = 0

IMTSINSTITUTE.COM
MATHEMATICS-I 8

a c -

Equating the determinant, we get

 -  ( a + b + c ) – 2 abc = 0
3 2 2 2

The characteristic equation of C is |C- I | = 0

- c b
|C-I|=0 ; c - a = 0
d a -

Expanding we get,
 -  ( a + b + c ) – 2 abc = 0
2

From I, II, III , we conclude that the matrices A, B, C have the same characteristic
equation and hence the same characteristic values.

1.2.5 Properties of characteristic Equations

Property : 1

Show that if A is a square matrix, the characteristic equation of A and A’ are


identical

Solution:

Let A = ( aij) i, j = 1, 2 , …. n

Then A - I =

a11 -  a12 … a1n


a21 a22 -  … a2n
A - I = a31 a32 a33 -  … a3n and
.
.
.
an1 an2 an3 … ann - 

a11 -  a12 … a1n


a21 a22 -  … a2n
A′ - I = a31 a32 a33 -  … a3n
.
.
.
an1 an2 an3 … ann - 

the value of a determinant is unaffected by interchanging of row into columns and columns
into rows.

|A-I| = | A′ -  I |

i.e., the scalar equation | A -  I | = 0, is the same


as the scalar equation | A′ -  I | = 0

Hence the characteristic equations of a A and A’ are identical.

IMTSINSTITUTE.COM
MATHEMATICS-I 9

Property : 2
-1
Show that the two matrices A and P A P have the same characteristic roots.

Solution
-1
Let B = P AP
B-I = P AP-I
-1

P [A -  I] P
-1
=

B-I | P | |A -  I | | P |
-1
=
|A - I | | P | | P |
-1
=
|A - I | | P | I
-1
=
= |A - I |

| A -  I | = 0  |B-  I | = 0

The characteristic roots of A and B =


-1
P A P are the same.

Property : 3

If and B are two non-singular matrices, Prove that AB and BA have the same
characteristic roots.

AB = I ( AB)
-1
= (B B ) AB
-1
= B (BA) B

From example same 5, it follows that AB and BA have the same characteristic
roots.

Property : 4

If the characteristic roots of A are 1, 2, 3, ….. n show that the characteristic roots
are 1 , 2 , ….. n ,
2 2 2 2
of A

Proof:

Since, 1 , 2, ….. n, are the characteristic roots of A, we have


|A - r I | = 0 , r = 1, 2, ….. n,

Then (A - r I ) (A + r I )

A - r I
2 2

 | A - r I | = | A - r I | | A + r I | = 0
2 2

 r ( r = 1, 2, … n) are the characteristic roots of A


2 2

Property : 5

If 1 , 2, ….. n, are the characteristic roots of A, the 1 1 1


1 , 2, ….. n
-1
are the characteristic roots of A

IMTSINSTITUTE.COM
MATHEMATICS-I 10

Solution

Since r ( r = 1, 2, … n) is a characteristic roots of A, we have

|A - r I | = 0

|A - r A A | = 0
-1

- r A A – _1_ I
-1
=0
r

( - r ) | A | A – _1_ I
n -1
=0
r

Since A is non-singular, | A |  0 and r  0


-1
A – _1_ I =0
r

the 1 1 1
1 , 2, ….. n are the characteristic roots of A
-1

Property : 6

cos   sin  
Find the characteristic roots of the orthogonal matrix  sin  cos  
and verify that they

are of unit modulus.

Solution :
cos   sin  
A=  sin  cos  

cos     sin  
| A - I | =  sin 
 cos    

 cos    + sin 
2 2
=

The characteristic equation is | A - I | = 0

 cos    + sin  = 0
2 2
i.e.

2 – 2  cos  + 1 = 0

2 cos   4 cos 2   4
=
2

IMTSINSTITUTE.COM
MATHEMATICS-I 11

= cos   i sin 

 the characteristic roots are cos   i sin 

=||= | cos   i sin  | = cos 2   sin 2  = 1

Example 7

If A is a skew Symmetric Matrix of order n, show that | A -  I | = ( - 1) | A +  I| .


n

Hence show that if  is a characteristic roots of A, then -  is also a characteristic root.

Solution :

If a is skew symmetric matrix, then A'  A

|A-I| = |A-I| '


= | A ' ( I ) ' |
= |  A  ( I ) | since I '  I
( - 1 ) |  A  ( I ) |
n
=

If  is a characteristic roots of A, then |A -  I| = 0

Then (1) gives | A  ( I ) | = 0

-  is also a characteristic root of A

0 c b 

Show that the Matrix c 0 a  satisfies Cayley – Hamilton’s theorem

 b a 0 
Solution

By Cayley- Hamilton’s theorem


A2  A  7 I  0

Divide f ( )by 2  5  7

|A -  I | X = 0
i.e. |A - I | X = 0

2 2 x1
i.e. 0
2 2 x2

2 x1  2 x2  0
2 x1  2 x2  0

IMTSINSTITUTE.COM
MATHEMATICS-I 12

Since x1 = k, x2 = - k satisfies the equation for all values of k. Then the characteristic
1
vector corresponding to  = 1, is X1  k  
 1

 c 2  b 2 ab ac 
 
  ac c 2  a 2 bc 
 ac bc b 2  a 2 

 c 2  b 2 ab ac  0 c b 
 
A3  A2 A   ac c  a2 2
bc   c 0 a 
 ac bc b 2  a 2   b  a 0 

 0 c ( a 2  b 2  c 2 ) b( a 2  b 2  c 2 ) 
 
=  c(a  b  c ) a(a 2  b 2  c 2 ) 
2 2 2
0
 b(a 2  b 2  c 2 ) c(a 2  b 2  c 2 ) 0 
 
 0 c b 
 (a 2  b 2  c 2 )  c 0  a 
 b a 0 

= ( a 2  b 2  c 2 ) A

=> A3  (a 2  b 2  c 2 ) A  0
Example 8
1 1 3

Find the characteristic equation of A = 5 2 6  and show that the matrix A satisfies

 2 1 1
the equation.

Solution:

The characteristic equation is A  I  0

1   1 3 
i.e.
 5 2 6  = 0

 2 1 3   

(1   )  (2   )(3   )  6   5(3   )  12   5  4  2   0


(1   )( 2   )  5  3  6  3  0
i.e.
 2     3   2  5  3  6  3  0
 3  0or  3  0

IMTSINSTITUTE.COM
MATHEMATICS-I 13

We will show that the matrix satisfies the characteristic equation. We have to verify that
A3  0

 1 1 3  1 1 3 
A   5 2 6   5 2 6 
2

 2 1 3  2 1 3

0 0 0

= 3 3 9 

 1 1 3

 0 0 0  1 1 3 
A   3 3 9   5 2 6 
3

 1 1 3  2 1 3

0 0 0 
=
0 0 0 
 
0 0 0 

 A3  0

Example 9

Use Cayley-Hamilton theorem to express 2 A5  3 A4  A2  4 I as a linear polynomial in


 3 1
A when A =  1 2 
 
Solution :

The characteristic equation of A is |A-  I| = 0

3   1 
i.e.  1 2    = 0
 
(3   )(2   )  1  0
 2  5  7 I  0
By Cayley-Hamilton Theorem
2
λ –5λ+7I

Consider the polynomial


f ( )  2 2  3 4   2  4
Divide f ( )by 2  5  7
3 2
(2λ – 7λ + 21λ +57)
2 2 4 2
( λ +5λ + 7) 2λ – 3λ + λ – 4
2 5  10 4  14 3

IMTSINSTITUTE.COM
MATHEMATICS-I 14

------------------------
7 4  14 3   2  4
7 4  35 3  49 2
------------------------
21 3  48 2  4
21 3  105 2  147
---------------------------
57 2  147  4
57 2  285  399
-----------------------
138  403

 2 2  3 4  4)  (2 2  5  7)(2 3  7 2  21  57)  138  403


 A5  3 A4  A2  4 I
A2  5 A  7 I
2 A3  7 A2  21A  571)  138 A  403I
 0  138 A  403I
 138 A  403I
Which is linear Polynomial is A.
Example 10

3 2
Find the characteristic vectors of the matrix 2 3  0
 
Solution:

The characteristic Equation is


3   2 
 2 0
 3   

(3   ) 2  4  0 or  2  6  5  0
(  1)(  5)  0
   1,5

The characteristic roots are 1, 5. The characteristic vector of A corresponding to


the characteristic root  =1,

(A  I)X  0
(A  I)X  0
 2 2  x1
2 2 x  0
i . e.
  2
2 X1  2 X 2  0
2 X1  2 X 2  0
Since X 1  k , X 2   k satisfies the equation to all the values of k.

IMTSINSTITUTE.COM
MATHEMATICS-I 15

1
The characteristic vector corresponding to  = 1, is k  
 1

 2 2  x1
When  = 5, we have  2 2  x  0
  2

2 X 1  2 X 2  0
2 X1  2 X 2  0

Here X 1  k , X 2  k satisfies the equation for all the vectors of k, the characteristic
vector corresponding to  = 5 is

1
X2  k  
1

SELF ASSESSMENT - II
1 0 2

1. Find the characteristic Equation of A = 0 2 1

 
 2 0 3 
…………………………………………………………………………………………

…………………………………………………………………………………………

…………………………………………………………………………………………

Ans:  3  6 2  7  2  0
1 0 3 

2. Verify Cayley – Hamilton theorem for the matrix 2 1 1

 
1 1 1 
…………………………………………………………………………………………

…………………………………………………………………………………………

…………………………………………………………………………………………

1.4 Rank of Matrix

The number r is said to be the rank of Matrix A if

(i) A possesses atleast one minor of order r which does not vanish
(ii ) Every minor of A of order ( r + 1) and higher orders vanish

In other words the rank of a matrix is the order of any highest order non vanishing
minor of the matrix. The rank of a matrix of A is denoted by  ( A)

Note 1 : If I is the unit matrix of order n, its rank is n


Note 2 : If A is any matrix of order m  n its rank is  m and n

IMTSINSTITUTE.COM
MATHEMATICS-I 16

Note 3 : The rank of A is the same as the rank of the transpose of A


Note 4 : The rank of a null matrix is taken as zero.

Example : 1
Consider the matrix

1 2 3

A= 2 4 6

 
 3 1 2 

Here the highest minor is of order 3. But |A| = 0

  ( A)  3

2 4
3 1  0
nd
But the 2 order
 
The rank of A = 2

1 2 3

Consider A = 2 4 0

 
 3 6 9 
Hence |A| = 0

Also every minor of order 2 vanish. But there exists non-zero minors of order 1

  ( A)  1

1.4.1 Elementary Transformation

In some cases the evaluation of determinant of matrices of highest order may not be simple.
In such cases we can obtain easily the rank of a matrix using elementary transformations.
An elementary transformation on a matrix is an operation of any one of the following three
types.

(i) The interchange of any two rows ( or column )

(ii) The multiplication of the elements of any row ( or column) by a non-zero scalar

(iii) The addition to the elements of any row ( or column) the scalar multiples of the
corresponding elements of any other row ( or column )

1.4.2 Equivalent matrices

Definition :

Two matrices A and B of the same order are said to be equivalent if one can be
obtained from the other by the application of a finite chain of elementary transformations.
Symbolically we write A – B and read it as A equivalent B

Example 1

Find the rank of the matrix

IMTSINSTITUTE.COM
MATHEMATICS-I 17

 3 1 2 

A = 6 2 4

 
 3 1 2 

Solution :
 3 1 2 
0 0 0  R1, R2  2 R1 , R3  R1
 
0 0 0 
Since all minors of order 3 and order 2 vanish  ( A)  3 ,  ( A)  2

There exists non-zero minors of order 1   ( A)  1

Example 2

Find the rank of

1 2 3 4

A= 2 4 6 8 

 1 2 3 4 
Solution:

1 2 3 4
A =
2 4 6 8 R2  2 R1 , R3  R1
 
 1 2 3 4 
  ( A)  1

Example 3

Find the rank of the Matrix

1 a b 0
0 c d 1 
A=  If a, b, c, d are all different
1 a b 0
 
0 c d 1

Solution:

1 a b 0 R1
0 c d 1 R2
A= 
0 0 0 0  R3  R1
 
0 0 0 0  R4  R2

  ( A)  2

IMTSINSTITUTE.COM
MATHEMATICS-I 18

Example 4

Find the rank of the Matrix

1 1 1
A=
a b c  If a, b, c, d are all different

 a 2 b2 c 2 

Solution:

|A| = (a-b) (b – c ) ( c – a )
Case 1

Suppose a, b,c are all different

Then |A|  0 ,  ( A)  3

Case 2
rd
Suppose two of a, b,c are equal and the 3 is different

i.e. Say a = b,  c
then |A| = 0, since 2 columns are identical

But
b c
b2 a2

1 1
= bc = bc ( c – b )  0
b a

  ( A)  2

Case 3:

Suppose all a, b, c are equal

i.e. a = b = c

1 1 1
 
then
a a a
 a2 a2 a 2 

In this case | A| = 0

Example 5

IMTSINSTITUTE.COM
MATHEMATICS-I 19

Find the Rank of

1 7 3 3

A = 7 20 2 25

 
 5 2 4 7 
Solution:

1 7 3 3 R1

A ~ 7 20 2 25
 R2
 
 0 33 11 22  R3  5 R1

1 7 3 3 R1
 
A ~ 0 69 23 46 R2  7 R1
 
0 33 11 22  R3

 1 2 0 0
 
- 0 0 23 46 C1, C2 + 3 C3, C3 + C4, C4 + 3C1
 
0 0 11 22 

 1 0 0 0
 
- 0 0 23 0 C1, C2 – 2C1, C3, C4 - 2C3
 
0 0 11 0 
rd
All 3 order minors vanish

  ( A)  3

But,

1 0 
  = 23  0,   ( A)  2
 0 23 

Example 6 :

Find the Rank of

IMTSINSTITUTE.COM
MATHEMATICS-I 20

1 2 1 2
1 3 2 2 
A= 
2 4 3 4
 
3 7 4 6

Solution:

1 2 1 2 R1
0 1 1 0 R2  R1
A~ 
1 0 0 2 R3  R2
 
0 0 1 0  R4  ( R2  R4 )

1 2 1 2  R1
0 1 1 0  R2
A~ 
1 0 0 2  R3  R2
 
0 0 1 0  R4

1 2 1 2  R1  R3
0 1 1 0  R2
A~ 
1 0 0 2 R3
 
0 0 1 0 R4
1 2 1 2
0 1 1 0 
A~  C1 , C2 , C3 , C4  2C1
1 0 0 0
 
0 0 1 0

0 1 0 0  R1  R2
0 1 0 0  R2  R4
~ 
1 0 0 0 R3
 
1 0 0 0 R4

0 0 0 0 R1  R2
0 1 0 0  R2
~ 
1 0 0 0 R3
 
0 0 1 0 R4

 ( A)  4 since A = 0

0 1 1
 
Consider the Minor 1 0 0 = 1  0
 
0 0 1

IMTSINSTITUTE.COM
MATHEMATICS-I 21

  ( A)  3

Self Assessment - III

Find the Rank of the following Matrices

1 3 4 3
(1)
 3 9 12 9 
 
 1 3 4 1

………………………………………………………………………………………..

………………………………………………………………………………………..

………………………………………………………………………………………..
Answer: 1

1 3 4 3 
(2 )
3 9 12 4 
 
1 3 4 1 
………………………………………………………………………………………..

………………………………………………………………………………………..

………………………………………………………………………………………..
Answer 2

1.4 Test of Consistency of linear equation

Working rule for finding the solution of the equation AX = B.

Suppose we are given m equations and n unknowns.

Step (i) Write down the coefficient Matrix A

Step ( ii) Write down the augmented matrix [ A, B]

Step ( iii) Apply elementary transformation to find the ranks of A and [A,B]

The following situations may arise

Case I

Rank A < Rank [A,B]

In this case equations are inconsistent


i.e. They have no solution

Case II
Rank A = Rank [A,B] = r (say )

In this case the equations are consistent

IMTSINSTITUTE.COM
MATHEMATICS-I 22

Ie. They possess a solution


If r = n then the solution is unique.
If r < n then there are infinite number of solution.

Example :1

Show that the system of equation


3x – 4y = 2
5x + 2y = 12
- x + 3y = 1 are consistent

Solution:
 3 4  2
A= 5
  12 
 2 ; B=  
 1 3   1 
 
3 4 2
(A,B) =
5 2 12 

 1 3 1 

 0 5 5  R1  3R3
-
 0 17 17  R  5 R
  2 3

 1 3 1  R3

5
R  R2
0 0 0 17
 
-
 0 17 17  R2
 1 3 1  R3
We note that P (A) = 2, and  (A,B) = 2

Hence the given system of equation is consistent and therefore possesses a


solution.

Example : 2

x  3 y  8 z  10
Show that the system of equations 3 x  y  4 z  0 are consistent and solve them
2 x  5 y  6 z  13

Solution
1 3 8

A= 3 1 4 

 2 5 6 

IMTSINSTITUTE.COM
MATHEMATICS-I 23

1 3 8 10  R1
(A,B)

= 3 1 4 0  R2

 2 5 6 13  11
R3  R2
10

= Here  (A) = 2,  (A,B) = 2

 The given system of equations is consistent.


Since,  (A), 2 < 3 there are infinite number of solutions.

x  3 y  8 z  10
 10 y  20 z  30
y  3  2z

Where z is a variable.

i.e. x = -1 +2z

Take z = k, the solutions are

x = - 1 + 2z
y = 3 – 2k
z = k where k is any arbitrary value.

Example 3

Show that the equation

x yz 6
x  2 y  3 z  14 are consistent and solve them
x  4 y  7 z  30

Solution:
1 1 1 
 
A= 1 2 3
 
1 4 7 
 
The Augmented matrix is

1 1 1 6 

(A, B) = 1 2 3 14

 
1 4 7 30 

IMTSINSTITUTE.COM
MATHEMATICS-I 24

1 1 1 6  R1
 
- 0 1 2 8 R2  R1
 
0 3 6 24  R3  R1

1 1 1 6  R1

- 0 1 2 8
 R
  2

0 0 0 0  R3  3R2

We note that  ( A)  2 and also


 ( A, B )  2

  ( A)  2 =  ( A, B )  2
Hence the given equations are consistent, But,   ( A)  2 =  ( A, B )  2 <3

 there are infinite number of solutions for the given equations. Also,
x yz 6
y  2z  8
y  9  2z
x  6  (8  2 z )  z
x  z2
Taking z = k where k is arbitary the solutions are

x  k 2
y  8  2k
zk
Example : 4

Show that the equation

x  2y  z  3
3x  y  2 z  1 are consistent and solve them.
x  y  z  1

Solution

1 2 1

A = 3 1 2

 
1 1 1 

IMTSINSTITUTE.COM
MATHEMATICS-I 25

1 2 1 3 
3 1 2 1 
(A,B) =  
1 1 1 2 
 
1 1 1 1

1 2 1 3  R1
0 7 5 8 R  R
-   2 3

0 6 5 4  R3  2 R1
 
0 3 2 4  R4  R1

1 2 1 3  R1
0 1 0 4  R  R
-   2 3

0 6 5 4  R3
 
0 3 2 4  R4

1 2 1 3  R1
0 1 0 4  R2
-  
0 0 5 20  R3  6 R2
 
0 0 2 8  R4  3R2

1 2 1 3  R1
0 1 0 4  R
-   2
0 0 1 4  R3/ 5
 
0 0 1 4  R4 / 2

1 2 1 3  R1
0 1 0 4  R
-   2

0 0 1 4  R3
 
0 0 0 0  R4  R3

   2 =  ( A, B)  3

 The given system of equations is consistent and has a unique solution


 the solution is z = 4
y=4
x + 2y – z = 6
x = -1
x = -1, y = 4, z = 4

SELF – ASSESSMENT IV

Test of Consistency and hence solve the equation:

IMTSINSTITUTE.COM
MATHEMATICS-I 26

x  2 y  3z  2
(i) 2 x  3z  3
x yz

………………………………………………………………………………………..

………………………………………………………………………………………..

………………….……………………………………………………………………..

Answer : k = - ½ , K = -4, Y = ½ where K -6 where z = k

1.5 Relation between the roots and coefficients of an Equation

Let 1, 2, … n be the roots of the equation

x n  p1 x n 1  p2 x n  2  ...  pn  0 … (1)

The equation corresponding to the roots 1, 2, … n is

(x  α1 )(x  α 2 ) . . . (x  α n )  0
x n   1 x n 1   1 2 x n  2  1 2 3  x n 3

. . .  (-1)n 1 2 ... n  0 …(2)

Comparing equations (1) & (2) we get,

1   p 1

1 ,  2  p2 1 ,  2 3   p 3
.
.

1 2 ... n  (1) n p n


If Sr is defined as the sum of the products of the roots of the equation taken r at a
time we have the following relations between the roots and coefficients.

S1   p1
S 2  (1) 2 p2
S3  (1) 3 p3

S n  (1) n pn

Note : If the given equation is

IMTSINSTITUTE.COM
MATHEMATICS-I 27

a0 x n  a1 x n 1  a2 x n  2  ...an 1 x  an  0 then divide the equation by a0

a1 n 1 a2 n  2 a n 1 a
xn  x  x  ...  x n 0
a0 a0 a0 a0

a1
Then S1  
a0
a2
S2  
a0
a3
S3  
a0

an
S n  (1) n
a0

Note 1:

(1) If , ,  are the roots of the equation ax +bx +cx+d =0 then


3 2

b
S1         .
a
c
S 2        .
a
d
S3     .
a

Note 2:

(2) If , , ,  are the roots of the equation x +px +qx +rx+s=0 then.
4 3 2

S1      p

S2     q

S3      r

S 4    s

Example 1

3 2 3
Show that if the roots of the equation x +px +qx+r=0 are in A.P. then 2p - 9pq + 27r
= 0.

Solution

IMTSINSTITUTE.COM
MATHEMATICS-I 28

Let the roots of the equation x +px +qx+r=0 be -, ,+ .


3 2

Sum of the roots is  -  +  +  +  = -p


p
i.e. 3   p or   
3

Since  is a root of the given equation it has to satisfy the equation x +px +qx+r=0.
3 2

3 2
 p  p  p
    p    q    r  0
 3  3  3
p 3 p 3 pq
   r 0
27 9 3
 p 3  3 p 3  9 pq  27 r  0
2 p 3  9 pq  27 r  0

Example 2

3 2
Solve the equation x -12x +39x-28= 0 whose roots are in A.P.

Solution

Let  - ,,  + be the roots


Then  -  +  +  +  = 12
i.e. 3 = 12 or = 4

4 is a root of the given equation. Let us remove this root by synthetic division.

1 -12 39 -28

0 4 -32 28

1 -8 7 0
2
The other roots are given by x – 8x + 7 = 0
(x – 1) (x - 7) = 0
x = 1 or x = 7

 The roots of the given equation are 1, 4, 7

Example 3

4 3 2
Solve the equation x -2x -21x +22x+40=0 given that the roots are in A.P.

Solution

IMTSINSTITUTE.COM
MATHEMATICS-I 29

4 3 2
x -2x -21x +22x+40=0

Let the roots be , , , 

Since they are in A.P.

+=+ …(1)

 x -2x -21x +22x+40=0


4 3 2

= (x-) (x-) (x-) (x-)

= [x -x) ( + ) +  ] [x -x (+)x+]
2 2

3
Equating the coefficients of x
+++=2
2(+)=2 [From (1)]
+=1 …(2)

2
Equating the coefficient of x

++(+)(+)= -21
++1=-21
+=-22
Equating the coefficient of x …(3)
(+)+(+)=-22
+=-22
Equating the constant term
=40 …(4)

By inspection from (4) & (5)


=-20
=-2
+=1, =-2 =-1, =2
+=1, =-20 =-4, =5
The roots are -4, -1, 2, 5

Example 4

4 3 2
If the sum of the two roots of the equation x +px +qx +x+ s=0 equals the sum of
3
the other two, prove that p +8r= 4pq

Solution

IMTSINSTITUTE.COM
MATHEMATICS-I 30

Let ,, ,  be the roots such that


+=+ ……(I)

Then x 4  px 3  qx 2  rx  s  ( x   )( x   )( x   )( x   )
= x – ( - )x + ) ( x –(+ )x + 
2 2

3
Equating the coefficient of x
 + ++ = - p … (1)
2
Equating the of efficient of x
(+) ( + ) + +  = q … (2)

Equating the coefficient of x


-  ( +) -  (+) = r
 (+)+  (+) = - r … (3)

Equating the constant term,


 = s

Using (I) in (1) we get 2 (+) = - p … (4)


p p
or        
2 2

p2
From (2),     q  … (5)
4

Using I in (3)

αβ  γδ    p   r
 2
2r
   
p
From (5) and (6)

p 2 2r
q 
4 p
4 pq  p 3  8r
or p 3  8r  4 pq

Example 5

Find the condition that the roots of the equation ax 3  3bx 2  3cx  d  0 are in
G.P.

IMTSINSTITUTE.COM
MATHEMATICS-I 31

Solution


Let the roots be ,  , 

 d
Then . .  
 a
d
i.e. 3   …(1)
a

Since  is a root of the equations

ax 3  3bx 2  3cx  d  0,
a 3  3b 2  3c  d  0
d 
a   3b.  3c  d  0
2

 a 
 d  3b 2  3c  d  0
3 b  c   0
Since  0, b+c = 0
or b = -c
b  = -c
3 3 3
Cubing,

d 
b3    c
3

 a 
d 
b3    c or b d  c a
3 3 3

 a 
This is the required condition.

Example 7

3 2
Solve the equation x -4x -3x+18=0 given that two of its roots are equation.

Solution

Let the roots of the equation

x 3  4 x 2  3 x  18  0 be  ,  ,  .

Then 2+=4

 +2=-3
2

 =-18
2

IMTSINSTITUTE.COM
MATHEMATICS-I 32

From (1), = 4- 2

Substituting this in (2)  + 2 (4-2)= - 3


2

 2  8  4 2  3

3 2  8  3  0

(3  1)(  3)  0

1
   or 3
3

1 14
When    ,  
3 3

When   3,   2

But  2   -18 is not satisfied by

1 14
  , 
3 3

 The roots are 3, 3, -2.

Example 8

Solve the equation x 4  2 x 3  4 x 2  22 x  40  0 given that its roots are in A.P.

Solution

Let the roots be   3 ,    ,    ,   3

Sum of the roots is

  3             3   2

1
4  2 or   
2

Also (  3 ) (   ) (   ) (  3 )  40
( 2   2 ) ( 2   2 )  40

IMTSINSTITUTE.COM
MATHEMATICS-I 33

1 2  1 
     9   40 
2

4  4 
1  4 1  36   640
2 2

144  2  40  2  639  0

40  368064

288
40  608

288
648 568
 
288' 288

= 648 - 568
288 , 288
9 - 71
4 , 36
9 3
When 2   
4' 2

1 3
When     the roots are -5, -2, -1, 4.
2' 2

1 3
Where      the roots are 4, -1, -2, -5.
2' 2

These roots satisfy the given equation and hence, -5,-2, -1, 4 are the only roots of
the given equation.

Example 9

Solve the equation x 4  2 x 3  25 x 2  26 x  120  0 given that the product of two


of its roots is 8.

Solution

Let the roots be , , , ; given that


=8

x 4  2 x 3  25 x 2  26 x  120  ( x   )( x   )( x   )( x   )

IMTSINSTITUTE.COM
MATHEMATICS-I 34

 [x 2  (   ) x   ][ x 2  (   ) x  y ] ...(1)

2
Equating the co-efficient of x ,

 +  +  +  = -2 …(1)

2
Equation the co-efficient of x ,

++(+)(+)= -25 …(2)

Equating the co-efficient of x

- [(+)+(+)] = -26 …(3)

Equating the constant term,

=120 …(4)

120
From (4), y   15
8

From (3), 15(   )  8(   )  26

(1) x 8 : 8(   )  8(   )  16

Subtracting

7(   )  42

(   )  6

From (1),     8

From I,

( x 2  6 x  8)( x 2  8 x  15)  0

i.e. ( x  2)( x  4)( x  3)( x  5)  0

the roots are 2, 4, -3, -5.

Example 12

4 3 2
Solve the equation 15x -8x – 14x + 8x -1 =0 given that the roots are in H.P.

IMTSINSTITUTE.COM
MATHEMATICS-I 35

Solution

4 3 2
15x -8x -14x +8x-1 = 0

If the roots of the equation are in H.P. then the roots of the equation
4 3 2
x -8x +14x +8x-15=0 are in A.P.
Let the roots be -3, -, +, +3
Sum of the roots is 4 = 8
 =2

Product of the roots is


2 2 2 2
( - ) ( -9 ) = -15
2 2
(4- ) (4-9 ) = -15
4 2
9 - 40 + 31 = 0
4 2 2
9 -9 -31 + 31 = 0
2 2 2
9 ( -1) -31 ( -1) = 0
2 2
( -1) (9 -31) = 0

31
β  1 or 
3

When =2, =1, roots are -1,1,3,5. When =2, = -1 roots are 5, 3, 1, -1 Which are
the same as the roots given by =2, = 1. These roots satisfy the given equation. Hence

31
these are only possible roots of the given equation and hence β is inadmissible.
3

SELF – ASSESSMENT – 5

1) Solve the value of k so that the roots of the equation

2 x 2  6 x 2  5 x  k  0 are in A.P
………………………………………………………………………………………..

………………………………………………………………………………………..

………………………………………………………………………………………..

3 2 1
Answer , ,10,
2 3 10

2. Solve the equation x  x  16 x  20  0 given that the difference between two


3 2

ratio is 7
………………………………………………………………………………………..

IMTSINSTITUTE.COM
MATHEMATICS-I 36

………………………………………………………………………………………..

………………………………………………………………………………………..

Answer ( 2, -2, 5)

1.6 Imaginary and irrational roots

Theorem

In a polynomial equation with real co-efficient imaginary roots occur in conjugate


pairs.

Proof

Let f(x) = 0 be a polynomial equation with real co-efficient.

Let  + i where  and  are real be an imaginary root of the equation f(x)=0.
We will now prove that  - i is also a root of the equation.

Now x  α  iβ x  α - iβ   x  α  iβ x  α  iβ 


= (x-) + 
2 2

which is a quadratic polynomial with real co-efficient.

Let us divide the polynomial f(x) by (x-) +  . Let the quotient be  (x) and the
2 2

remainder be Ax + B.

Then f (x) = [(x-) +  ]  (x) + AX + B


2 2

By hypothesis  + i is a root of f(x) = 0


 f (+i) = 0
f (+ i) = 0 + A(+ i) + B = 0
Equating real and imaginary parts we get
A + B=0 … (1)
A = 0 … (2)
Since  + i is an imaginary root   0 and therefore from (2), A =0
Consequently from (1), B = 0
Hence the remainder =Ax +B =0

This means (x-) +  is a factor of f (x).


2 2

In other words,  + i and  - i are the roots of f (x)=0.

Hence in a polynomial equation with real co-efficient imaginary occur in conjugate

pairs.

IMTSINSTITUTE.COM
MATHEMATICS-I 37

Example 2

4 2
Solve the equation x – 11x +2x +12 = 0 given that 5  1 is a root.

Solution

In an equation with irrational roots occur in conjugate airs. Since 5  1 is a root.


 5  1 is also a root. The real factor corresponding to these roots is

( x  1  5 )( x  1  5 )  ( x  1) 2  5
= ( x 2  2 x  4)

x4 -11x 2 + 2x + 12
( x 2  2 x  4) x2  2x  3
x4 +2x 3 - 4x2

- 2x3 -7x 2 + 2x
- 2x3 -4x 2 + 8x

- 3x2 - 6x + 12
- 3x2 - 6x + 12

0
 The other roots are given by

2
x -2x-3 = 0
(x-3) (x+1) =0
x=3, -1

 The roots of the equation are – 1 - 5 , - 1 + 5 , - 1, 3.

Example 3

Given that  2   7 is a root of the equation x4+2x2-16x+77=0 solve it completely

Solution

 2   7  2  i 7

Since  2  i  7 is a root,  2  i 7 is also a root.

The real factor corresponding to these roots is

( x  2  7i )( x  2  7i )

 ( x  2) 2  7  x 2  4 x  11

IMTSINSTITUTE.COM
MATHEMATICS-I 38

( x 2  4 x  11) x4 +2x 2 - 16x + 77 x  2 x  3


2

x4 +4x 3 +11x2

- 4x3 -9x 2 - 16x


- 4x3 -16x 2 - 44x

7x2 +28x + 77
7x2 +28x + 77

 The other roots are given by x 02  4 x  7  0

4  16  28 4  3i
x   2i 3
2 2

The roots are  2  i 7 , 2  3i

Example 4

Solve the equation x 4  5 x 3  4 x 2  8 x  8  0 given that 1 5 is a root.

Solution

1 5 is a root. 1 5 is also a root.

Let the other roots be , .

Sum of the roots = 1 5 1 5      5

+= 3

Product of the roots = (1  5 )(1  5 )  8

 4  8
  2

 &  are the roots of x 2  x(   )    0

x 2  3x  2  0

( x  1)( x  2)  0

IMTSINSTITUTE.COM
MATHEMATICS-I 39

x  1,2

 roots of the given equation are 1  5 , 1  5 , 1, 2

Self – Assessment - 6

1. Solve the equation x 4  4 x 3  8 x  35  0 given that 2  3i is roots


………………………………………………………………………………………..

………………………………………………………………………………………..

………………………………………………………………………………………..

Answer : - 1,  5 , 3, -1

1.7 Transformation of equation

If 1, 2,….n are the roots of the equation f(x)=0, then forming the equation whose
roots are (1), (2),…. (n) is called transformation of the equation.

The relation between a root of f(x)=0 and a root y of the required equation is y=(x).
If x is eliminated between the equation f(x)=0 and y=(x), an equation in y is
obtained and this is the required equation whose roots are (1), (2),… (n).

Example 1

If , ,  are the roots of the equation x + px +qx + r = 0 form the equation whose
3 2

α 2  βγ β 2  γα γ 2  αβ
roots are , ,
α β γ

Solution

, ,  are the roots of

3 2
X + px + qx + r =0
++ = -p …. (2)
+  +  = q
 = -1 … (3)
We have to form the equation whose roots are

α 2  βγ β 2  γα γ 2  αβ
, , .
α β γ

IMTSINSTITUTE.COM
MATHEMATICS-I 40

α 3  αβγ α 3  r
Let y =  – =
2

α2 α2
x3  r
(i.e) y =
x2
3 2
X - yx + r =0
3 2
But x + px + qx + r = 0 ….(1)

2
Subtracting we get x (p+ y) + qx = 0
Since x  0, x (p+y) = - q
q
or x =
py
Substituting in the given equation (1) we get,

3 2
 q   q   q 
   p    q    r  0
py py py
3 2 2 2 3
r (p+y) – q (p+y) + pq (p+y) – q = 0
3 2 2 2 2 3 3
(i.e.) ry +y (3pr- q ) + y (3p r- q p) + rp - q = 0 is the required equation.

Example 2

If , ,  are the roots of the equation x +qx+r=0 find the equation whose roots are
3

     
 ,  ,  .
     
Solution

If , ,  are the roots of

x 3  qx  r  0
      0
      q
   r

  2  2
Let y   
  

 2  2
(    )  2y
2

 
  / 
 3  2  3  2r
 
 r

IMTSINSTITUTE.COM
MATHEMATICS-I 41

 3  2r
i.e. y 
r
i.e. x 3  ry  2r  0
But x 3  qx  r  0

Subtracting

ry  qx  r  0

r ( y  1)
x 
q

Substituting in the given equation we get,

r 3 ( y  1) 3 qr ( y  1)
 r 0
q3 q

r 3 ( y  1) 3  q 3 r ( y  1)  rq 3  0

r 2 ( y 3  3 y 2  3 y  1)  q 3 ( y  1)  q 3  0

r 2 y 3  3r 2 y 2  y (3r 2  q 3 )  r 3  2q 3  0

Example 3

, ,  are roots of x 3  2 x 2  3 x  3  0 . First let us form the equation whose roots


2 2 2
are    13
(  1) 2 (   1) 2 (  1) 2

Solution

, ,  are the roots of x 3  2 x 2  3 x  3  0 . First let us form the equation whose


  
roots are , , .
 1  1  1

 x
Let 
 1 x 1

y
 yx  y  x or x 
1 y

Substituting this in the given equation we get,

IMTSINSTITUTE.COM
MATHEMATICS-I 42

3 2
 y   y   y 
   2   3   3  0
 1  y   1  y   1  y 

y 3  2 y 2 (1  y )  3 y (1  y ) 2  3(1  y 3 )  0

y e  2 y 2 (1  y )  3 y (1  2 y  y 2 )  3(1  3 y  3 y 2  y 3 )  0

y3  5 y2  6 y  3  0

Let us now form the equation whose roots are

2 2 2
, ,
(1   ) 2 (1   ) 2 (1   ) 2

2
Let z   y2
(1   ) 2

2
Substitute z=y in (2)
zy  5 z  6 y  3  0
y ( z  6)  5 z  3

Squaring
y 2 ( z  6) 2  (5 z  3) 2

z ( z 2  12 z  36)  25 z 2  30 z  9
z 3  13 z 2  6 z  9  0
2 2 2
 Sum of the roots     13
(1   ) 2 (1   ) 2 (1   ) 2

Self – Assessment : 8

1.  ,  ,  are the roots of the equation x 3  qx  r  0 form the equation whose


If
roots are  ,  , 
2 2 2

………………………………………………………………………………………..

………………………………………………………………………………………..

………………………………………………………………………………………..

Ans: x3  2qx 2  q 2 x  r 2  0

IMTSINSTITUTE.COM
MATHEMATICS-I 43

UNIT – II

2.0 Introduction
2.1 Objectives
2.2 Successive Differentiation
2.2.1 SELF-ASSESSMENT QUESTION - I

2.3 n-th Derivative

2.3.1 SELF ASSESSMENT QUESTIN – II

2.4 LEIBNITZ THEOREM

2.4.1 SELF ASSESSMENT QUESTION - III

2.5 Euler’s Theorem on homogeneous functions

2.5.1 SELF ASSESSMENT QUESTION - IV

2.6 Curvature and Radius of Curvature

2.6.1 Cartesian Formula for Radius of curvature


2.6.2 Parametric formula for Radius of Curvature
2.6.3 SELF ASSESSMENT – V

IMTSINSTITUTE.COM
MATHEMATICS-I 44

2.0 Introduction

In this chapter, we have to deal with successive differentiation and understanding


the definition and applied in to the problem. It will lead up to the nth derivative.

2.1 Objective

To develop the skill in the successive differentiation

To create new idea to application oriented sum.

To understanding the way of approach for the nth derivative

2.2 Successive Differentiation


dy
If y is a function of x, its derivative will be some other function of x and the
dx
differentiation of this function with respect to x is called second derivative and is denoted by
d2y
dx 2

d  dy  d 2 y
i.e.  
dx  dx  dx 2

d3y
Similarly the third derivative is denoted by
dx3
d  d2y  d3y
i.e.  
dx  dx 2  dx 3

Thus if we differentiate y twice with respect to x, we get the second derivative. If y is


differentiated thrice with respect to x we get the third derivative.

Example 1

ax+b d2 y
If y = find
cx+d dx 2

Solution:
5 1 5 1 13 1
(i.e.) y  .  . 
32 2 x  1 16 2 x  1 32 2 x  3
5 (1) n (n !2n ) 5 (1) n (n !2n ) 13 (1) n ( n !2n )
 yn   
32 (2 x  1) n 1 16 (2 x  1) n 1 32 (2 x  3) n 1
5 1 5 1 13 1
= (-1) n n!2n [ . n 1
 . n 1
 . ]
32 (2 x  1) 16 (2 x  1) 32 (2 x  3) n 1

IMTSINSTITUTE.COM
MATHEMATICS-I 45

d 2 y 0  (ad  bc)(2)(cx  d )c

dx 2 (cx  d ) 4

acx  ad  acx  bc
=
(cx  d ) 2

ad  bc
=
(cx  d ) 2

d 2 y 0  (ad  bc)(2)(cx  d )c

dx 2 (cx  d ) 4

2c(ad  bc)
=
(cx  d )3

Example 2

If x = a(Cost + tSint)
d2y
y =a(Sint-tCost) find
dx 2
Solution

y =a(Sint-tCost)

dx
 a ( S int  tCost  S int)
dt
= atSint
x = a(Cost + tSint)

dx
 a ( S int  tCost  S int)
dt
= atcost

dy dy dt atS int
 .   tan t
dx dt dx atCost

d2y d dy d dy dt
2
 ( ) ( )
dx dx dx dt dx dx

d dt
= (tan t ).
dt dx

2 1
= sec t.
at cos t

sec3 t
=
at

IMTSINSTITUTE.COM
MATHEMATICS-I 46

Example 3

d2y
If y = acos5x+bsin5x show that  25 y  0
dx 2
Solution
y = acos5x+bsin5x
Differentiating with respect to x

dy
 5a sin 5 x  5b cos 5 x
dx
d2y
 25a cos 5 x  25b sin 5 x
dx 2
= -25(acos5x+bsin5x)
= - 25y

d2y
  25 y  0
dx 2
Example 5

If y = ( x  1  x 2 ) m show that (1+x2)y2+xy1-m2y=0

Solution
y = (x  1 x )
2 m

Differentiating with respect to x

dy 2x
 m( x  1  x 2 ) m 1.[1  ]
dx 2 1  x2
m( x  1  x 2 ) m 1[ 1  x 2  x]
=
1  x2
m( x  1  x 2 ) m
=
1  x2
my
=
1  x2

Cross multiplying and squaring we get,

dy 2
(1  x 2 )( )  m2 y 2
dx
d2y dy
(1  x 2 ) 2  x( )  m 2 y  0
dx dx
 (1  x ) y2  xy1  m 2 y  0
2

IMTSINSTITUTE.COM
MATHEMATICS-I 47

dy
Canceling, 2 we get,
dx

d2y dy
(1  x 2 ) 2
 x( )  m 2 y  0
dx dx
 (1  x ) y2  xy1  m 2 y  0
2

Example 5
-1 2 2 2
If y = (tan x) show that (1+x )y2+2x(1+x )y1=2

Solution
-1 2
y = (tan x)
Differentiating with respect to x,
2 tan 1 x
y1 
1  x2
2 -1
(1+x )y1 = 2tan x

2 2
Again differentiating (1+x )y2+y12x =
1  x2
2 2 2
= (1+x ) y2+2x(1+x ) y1=2

Example 6
m d2y 22 2
If y = sin x prove that sin x 2
= (m cos x-m)y
dx
Solution
m
y = sin x
Differentiating with respect to x
dy
 m sin m 1 x cos x
dx
d2y
2
 m(m  1) sin m  2 x cos 2 x  m sin m x
dx
2
Multiplying both sides by sin x,

d2y
 sin 2 x
dx 2
m 2 m 2
= m(m-1)sin xcos x-msin x.sin x
2 2
= m(m-1)ycos x-mysin x
2 2 2 2
= m ycos x-mycos x-mysin x
2 2 2 2
= m ycos x-my(cos x+sin x)
d2y
 sin 2 x 2
 m 2 y cos 2 x  my
dx
2
= (mcos x-m)y

2.2.1 SELF-ASSESSMENT QUESTION - I

d2y
1. Find
dx 2

IMTSINSTITUTE.COM
MATHEMATICS-I 48

x 1
y
x 1
……………………………………………………………………………………..

……………………………………………………………………………………..

……………………………………………………………………………………..
4
Ans:
( x  1)3

2. If xy  x  1 show that x y  2 xy  2 y  0
2 2

……………………………………………………………………………………..

……………………………………………………………………………………..

2.3 n-th Derivative

y yx
ax n ax
e ae

n
1 b
(a  b ) e ax cos(bx  c  n tan 1 )
2 2 2
(ax  b) 2 a

1 (1) n (n  1)!a n
(ax  b) 2 (ax  b) n  2
log(ax+b)
(1) n 1 (n  1)!a n
(ax  b) n
n n
sin(ax+b) a sin(ax+b+ )
2
coa(ax+b) n n
a cosx(ax+b+ )
ax
e sin(bx+c) 2
n
b
(a  b ) e ax sin(bx  c  n tan 1 )
2 2 2

ax a
e cos(bx+c) n
b
(a 2  b 2 ) 2 e ax cos(bx  c  n tan 1 )
a
Example 1
2x 1
Find the nth derivative of
(2 x  1)(2 x  3)

Solution

2x 1
Let y
(2 x  1)(2 x  3)

2x 1 A B
Let = 
(2 x  1)(2 x  3) (2 x  1) (2 x  3)

IMTSINSTITUTE.COM
MATHEMATICS-I 49

2x+1=A(2x+3)+B(2x-1)

1 1
Put x  2  A(4) or A =
2 2

3 1
Put x  2  B(4) or B =
2 2

1/ 2 1/ 2
y 
(2 x  1) (2 x  3)
1 1
(1) n 2n n ! (1) n 2n n !
 yn = 2 n 1
2
(2 x  1) (2 x  3) n
1 1
= (-1) n 2n 1 n ![ n 1
 ]
(2 x  1) (2 x  3) n 1

Example 2
x2  1
Find the nth derivative of
(2 x  1)(2 x  1)(2 x  3)
Solution

x2  1 A B C
let y    
(2 x  1)(2 x  1)(2 x  3) 2 x  1 2 x  1 2 x  3

2
x +1 = A(2x+1)(2x+3)+B(2x-1)(2x+3)+C(2x-1)(2x+1)

1 5 5
put x = ;  A(2)(4)  A=
2 4 32
1 5 -5
put x = - ;  B(2)(2)  B =
2 4 16
-3 13 13
put x = ;  C (4)(2)  C =
2 4 32

5 1 5 1 13 1
(i.e.) y  .  . 
32 2 x  1 16 2 x  1 32 2 x  3
5 (1) n (n !2n ) 5 (1) n (n !2n ) 13 (1) n ( n !2n )
 yn   
32 (2 x  1) n 1 16 (2 x  1) n 1 32 (2 x  3) n 1
5 1 5 1 13 1
= (-1) n n!2n [ . n 1
 . n 1
 . ]
32 (2 x  1) 16 (2 x  1) 32 (2 x  3) n 1

Example 3

x a
If tan 1 ( ) show that yn = (-1)n-1(n-1)!a-n sinn  sinn  where   tan 1 ( )
a x

IMTSINSTITUTE.COM
MATHEMATICS-I 50

Solution

x
let y = tan -1 ( )
a
2
a
y1  2
x  a2
1 1 1
= [  ]
2i x  ai x  ai
1 1 1
y n  (1) n 1 (n  1)![  ]
2i ( x  ai ) ( x  ai ) n
n

Put x = rcos  ; a = rsin 


1
 y n  (1) n 1 (n  1)![r  n (cos   i sin  )  n  r  n (cos   i sin  )  n ]
2i
1
= (1) n 1 (n  1)!r  n 2i sin n
2i
= (1) n 1 (n  1)!a  n sin n  sin n (  a = rsin )

Example 4
3 2
Find the nth derivative of sin xcos x

Solution
3 2
y = sin xcos x

(3sin x  sin 3 x) 1  cos 2 x


= [ ]
4 2
1
= [3sin x  sin 3 x  3cos 2 x sin x  sin 3 x cos 2 x]
8
1 3 1
= [3sin x  sin 3 x  (sin 3 x  sin x)  (sin 5 x  sin x)]
8 2 2
1
= [6sin x  2sin 3 x  3sin 3 x  3sin x  sin 5 x  sin x]
16
1
= [2sin x  sin 3 x  sin 5 x]
16
1 n n n
y n  [2sin( x  )  3n.sin(3 x  )  5n sin(5 x  )]
16 2 2 2

IMTSINSTITUTE.COM
MATHEMATICS-I 51

Example 5
5 4
Find the nth derivative of sin xcos x

Solution
5 4
Let y = sin xcos x let z = cosx+isinx
1 5 1 4
Consider (z- ) (z+ )
z z

1 1 1 1
(z- )5 (z+ ) 4 = (z 2 - 2 ) 4 (z- )
z z z z
4 1 1
= (z8  4z 4  6  4
 8 )(z- )
z z z
4 1 6 4 1
= z9  4 z 5  6 z  3  7  z 7  4 z 3   5  9
z z z z z
1 1 1 1 1
= (z9  9 )  (z 7  7 )  4(z5  5 )  4(z3  3 )  6(z  )
z z z z z
5 4
(2isinx) (2cosx) = 2isin9x-2isin7x-4(2isin5x)+4(2isin3x)+6(2isinx)
Canceling 2isinx,
5 4
Let y = sin xcos x

1
= [sin 9 x  sin 7 x  4sin 5 x  4sin 3 x  6sin x]
28
1 n n n
y n  8 [9n sin(9 x  )  7 n sin(7 x  )  4.5n sin(5 x  )
2 2 2 2
n n
+4.3n sin(3x  )  6sin( x  )]
2 2
2.3.1 SELF ASSESSMENT QUESTIN – II

1. If y  sin 1 x Prove that (1  x 2 ) y2  xy1  0

……………………………………………………………………………………..

……………………………………………………………………………………..

……………………………………………………………………………………..

IMTSINSTITUTE.COM
MATHEMATICS-I 52

2.4 LEIBNITZ THEOREM

If u and v are functions of x and n is a positive integer then


n
D (uv) = unv+nc1un-1v1+ nc2un-2v2+…+ ncrun-rvr+…+uvn
n
Where D (uv) standards for the nth derivative of uv.

Proof:

Let us prove this result by induction.

When n = 1, D(uv) = u1v+uv1 which is the product rule for the differentiating of the product
function uv. Therefore the result in true for n = 1.

Let us now assume the result to be true for n = m where m is a fixed integer.
m
(i.e.) D (uv) = umv+mc1um-1v1+ mc2um-2v2+…+ mcrum-rvr+…+uvm …..(1)

let us prove that the result in true for n = m+1

i.e. to prove,
m+1 m+1 m+1 m+1
D (uv) = um+1v+ C1umv1+ C2um-1v2+…+ Crum-r+1vr+…+uvm+1

To prove this let us differentiate (1) with respect to x.


m+1
D (uv) = (um+1v+umv1)+mc1(umv1+um-1v2)+mc2(um-1v2+um-2v3)+….+
m
Cr(um-r+1vr+um-rvr+1)+….+ (u1vm+uvm+1) …..(2)
m m m m m m
= um+1v+( C0+ C1)umv1+( C1+ C2)um-1v2+( C2+ C3)um-3v2+….+
m m
( Cr-1+ Cr)um-r+1vr+…+uvm+1
m+1 m+2 m+1
= um+1v+ C1umv1+ C1um-1v2+…+ Crum-r+1vr+…+uvm+1
m m m+1
( C r-1 + C r= Cr )

Which is (2). Therefore the result is true for n = m+1

Since the result is true for n=1, it is true for n=2, n=3 and so on. Therefore the result is
true for all positive integer values of n.

Example 1
2 5x
Find the nth derivative of x e

Solution
5x 2
Let us choose u = e and v = x

IMTSINSTITUTE.COM
MATHEMATICS-I 53

In 1 1 1 1
 I1     ... 
n! 2 3 4 n
I 1 1 1 1
 n  I1     ... 
n! 2 3 4 n
1 1 1 1
Then I n  n ![I1     ...  ]
2 3 4 n
d
But I1  ( x log x)  log x  1
dx
1 1 1
 I n  n ![log x  1    ...  ]
2 3 n
n 5x 2 n-15x n-2 5x
= 5 e x +nc15 .2x+nc25 e .2

n(n  1)
 e5 x .5n  2 [52 x 2  n  5  2 x  .2]
2
5x n-2 2
= e .5 [25x +10nx+n(n-1)]

Example 2
x
Find the nth derivative of e logx

Solution
x
Let us choose u = logx and v = e

dny
 un v  nc1un 1v1  nc2un  2 v2  ....  uvn
dx n

(1) n 1 (n  1)!
x
If u = e then un = e
x
If v = logx then vn 
xn

1 1 2! (1) n 1 (n  1)!
 yn  e x .log x  nc1e x .  nc2 e x .( 2 )  nc3e x . 3  ...  e x
x x x xn

1 1 2! (1) n 1 (n  1)!
 e x [log x  nc1.  nc2 2  nc3 . 3  ...  ]
x x x xn
Example 3

dn n 1 1 1
If I n  n
[ x log x] prove that In  n ![log x  1    ...  ]
dx 2 3 n
Solution

IMTSINSTITUTE.COM
MATHEMATICS-I 54

dn n
I n  n ( x log x)
dx
d n 1 d
= n 1 . ( x n log x)
dx dx
n 1
d 1
= n 1 [ x n .  n.x n 1 log x]
dx x
n 1
d d n 1
= n 1 ( x n 1 )  n n 1 ( x n 1 log x)
dx dx
= (n-1)!+nIn-1

In I n-1 1
Dividing by n!. - =
n ! (n  1)! n

Changing n into n-1, we get

I n-1 I n-2 1
- =
(n  1)! (n  2)! n-1
I n-2 I n-3 1
similarly, - =
(n  2)! (n  3)! n-2
.
.
I2 I 1
 1
2! 1! 2
Adding all these we get,

In 1 1 1 1
 I1     ... 
n! 2 3 4 n
In 1 1 1 1
  I1     ... 
n! 2 3 4 n
1 1 1 1
I n  n ![I1     ...  ]
2 3 4 n
d
But I1  ( x log x)  log x  1
dx
1 1 1
 I n  n ![log x  1    ...  ]
2 3 n
Example 4
-1 y x 2 2
If cos ( ) = nlog( ), prove that x yn+2+(2n+1)xyn+1+2n yn=0
n b
Solution

-1 y
cos ( )=n(logx-logn)
n

IMTSINSTITUTE.COM
MATHEMATICS-I 55

Differentiating with respect to x,

1 1 dy n
 . 
y2 b dx x
1
b2

1
dy n
 
b  y dx x
2 2

Squaring and cross multiplying

dy 2
x2 ( )  n 2 (b 2  a 2 )
dx
Differentiating with respect to x,
dy d 2 y dy dy
x 2 (2 2
)  ( ) 2 .2 x  n 2 (2 y )
dx dx dx dx
dy 2 2
Canceling 2 ,x y2+xy1+ny =0
dx
Applying leibnitz theorem to differentiate this further n times,
2 n n n 2
i.e.,[yn+2x + C1yn+1.2x+ C2yn.2]+[yn+1.x+ C1yn.1]+n yn=0
2 2 2
i.e., x yn+2+2nxyn+1+n yn-nyn+xyn+1+nyn+n yn=0
2 2
i.e., x yn+2+(2n+1)xyn+1+2n yn=0

Example 5

1 1

If y y
m m
 2 x prove that (x2+1)yn+2+(2n+1)xyn+1+(n2-m2)yn=0

Solution
1 1

ym  y m
 2x
Differentiating with respect to x,
1 1
1 m 1 1  1
y y1  y m y1  2
m m
1 1
y1 m 
i.e., (y  y )  2
m
my
Squaring and cross multiplying
1 1

y12 ( y m  y m 2
)  4m 2 y 2
1 1

y12 [( y m  y m 2
)  4]m 2 y 2

2 2 2 2
Y1 (4x -4) = 4m y
2 2 2 2
(x -1)y1 = m y

Differentiating with respect to x,


2 2 2
(x -1)2y1y2+y1 (2x)=m .2yy1

IMTSINSTITUTE.COM
MATHEMATICS-I 56

Canceling 2y1,
2 2
(x -1)y2+xy1-m y=0

Applying Leibnitz theorem to differentiate this further n times, we get

[( x 2  1) y2 ]n  [ xy1 ]n  [m 2 y ]n  0
2 2
Yn+2(x -1)+nc1yn+1.2x+nc2yn.2+(yn+1.x+nc1yn.1)-m yn=0
2 2 2
(x -1)yn+2+2nxyn+1+n yn-nyn+xyn+1+nyn-m yn=0
2 2 2
i.e., (x +1)yn+2+(2n+1)xyn+1+(n -m )yn=0

2.4.1 SELF ASSESSMENT QUESTION - III

 ax
1. If y  (1  x)2 e P.T . (1  x) yn 1  (n  2 x) yn  n yn 1  0
………………………………………………………………………………………………

………………………………………………………………………………………………

………………………………………………………………………………………………

2.5 Euler’s Theorem on homogeneous functions

If ‘f’ is a homogeneous function of degree ‘n’ in x and y then


f f
x y  nf
x y
Proof
Since f is a homogeneous function of degree ‘n’ in x and y we can write ‘f’ as
y
f  xn F ( ) ….(1)
x
Differentiating partially w.r.t x,
f y y y
 x n F 1 ( )( 2 )  nx n 1F ( )
x x x x
….(2)
f y y
x   x n 1F 1 ( )  nx n F ( )
x x x
Differentiating (1) partially w.r.t y
f y 1
 x n F ( )( )
y x x
f y
y.  yx n 1F 1 ( ) .....(3)
y x
Adding (2) and (3)
f f n y
x +y =nx F( )
x y x
f f
i.e., x +y =nf
x y

IMTSINSTITUTE.COM
MATHEMATICS-I 57

Note: The generalized form of Euler’s theorem for homogeneous function of degree ‘n’ in k
variables x1,x2,…,xk is

f f f f
x1  x2  x3  ....  xk  nf
x1 x2 x3 xk

Example 1
u u u
If u = (x-y)(y-z)(z-x) show that   0
x y z
Solution

u = (x-y)(y-z)(z-x)

u
=(y-z)(z-x)-(x-y)(y-z)
x

u
=(z-x)(x-y)-(z-x)(y-z)
y
u
=(x-y)(y-z)-(x-y)(z-x)
z

u u u
Adding   0
x y z

Example 2
2 z 2 z
x
If z = e (x cosy – y siny), show that  0
x 2 y 2

Solution
x
z = e (x cosy – y siny)
z x x
=e (x cosy – y siny)+e .cosy
x
2z x x
= e (x cosy – y siny)+2e cosy
x 2
x
= e (x cosy – y siny +2 cosy) ….(1)
z x
= e (-x siny –y cosy –siny)
y
2z x
=e (-x cosy + y siny –cosy – cosy)
y 2
x
= e (-x cosy -2 cosy + y siny)

Adding (1) and (2)

2 z 2 z
 0
x 2 y 2

Example 3

IMTSINSTITUTE.COM
MATHEMATICS-I 58

1 2 f 2 f 2 f
If show that   0
x2  y2  z 2 x 2 y 2 z 2

Solution
1

f  ( x2  y2  z 2 ) 2

f
3
1 
  ( x 2  y 2  z 2 ) 2 .2 x
x 2
3

 - x( x2  y 2  z 2 ) 2

2 f
5 3
3  
 x. ( x 2  y 2  z 2 ) 2 .2 x  ( x 2  y 2  z 2 ) 2
x 2
2
5 3
 
 3x ( x  y  z )
2 2 2 2 2
 (x  y  z )
2 2 2 2

 f
2 5 3
 
Similarly  3y 2 ( x 2  y 2  z ) 2 2
 ( x2  y2  z 2 ) 2
y 2

2 f
5 3
 
 3z 2 ( x 2  y 2  z 2 ) 2  ( x 2  y 2  z 2 ) 2
z 2

 f 2 f 2 f
2 5 3
 
Adding   = 3(x  y  z )( x  y  z )  3( x  y  z ) 2
2 2 2 2 2 2 2 2 2 2

x 2 y 2 z 2
3 3
 
= 3( x
2
y z ) 2 2 2
 3( x  y  z )
2 2 2 2

=0

Example 4

2 f 2 f 2 f 2
If f = x  y  z prove that
2 2

2
 =
x 2 y 2 z 2 f
Solution
1
f  (x  y  z )
2 2 2 2

f 1 2
1

 ( x  y 2  z 2 ) 2 .2 x
x 2
1

 x( x2  y 2  z 2 ) 2

2 f
3 1
1  
  x ( x 2  y 2  z 2 ) 2 .2 x  ( x 2  y 2  z 2 ) 2
x 2
2
 f
2 3 1
 
 - x (x  y  z )  (x  y  z ) 2
2 2 2 2 2 2 2 2

x 2
2 f
3 1
 
Similarly  - y 2
( x 2
 y 2
 z 2
) 2
 ( x 2
 y 2
 z 2
) 2
y 2
2 f
3 1
 
 - z2 ( x2  y2  z 2 ) 2
 ( x2  y2  z 2 ) 2
z 2

IMTSINSTITUTE.COM
MATHEMATICS-I 59

2 f 2 f 2 f
3 1
 
Adding  2  2 = - (x 2  y 2  z 2 )( x 2  y 2  z 2 ) 2  3( x 2  y 2  z 2 ) 2
x 2
y z
1 1
 
=  ( x2  y2  z 2 ) 2
 3( x 2  y 2  z 2 ) 2

1
 2
= 2 (x
2
 y2  z2 ) 2 =
f

Example 5

u u u
2 2 2
If u = x (y-z) + y (z-x) +z (x-y) show that   0
x y z

Solution
2 2 2
u = x (y-z) + y (z-x) +z (x-y)
u 2 2
= 2x(y-z) – y +z
x
u 2 2
= x –z +2y(z-x)
y
u 2 2
= 2x(x-y) –x +y
z

u u u
Adding   = 2[x(y-z)+y(z-x)+z(x-y)]=0
x y z

Example 6
2 z 2  z
2
3/2
If z = tan(y+ax) + (y-ax) , show that  a
x 2 y 2
Solution
3/2
z = tan(y+ax) + (y-ax)
z
1
3
 sec 2 ( y  ax).a  ( y  ax) 2 ( a )
x 2
 z
2 1
3 
 2 sec 2
( y  ax ) tan( y  ax ).a 2
 ( y  ax ) 2 2
a
x 2
4
1
3 
 a 2 [2 sec 2 ( y  ax) tan( y  ax)  ( y  ax) 2 .....(1)]
4
z
1
3
 sec 2 ( y  ax).a  ( y  ax) 2
y 2
2 z
1
3 
 2 sec 2 ( y  ax) tan( y  ax)  ( y  ax) 2 .....(2)
y 2
4
2 z 2  z
2
From (1) and (2),  a
x 2 y 2
Example 7

u u u
If u = log(tanx + tany + tanz) prove that sin 2 x  sin 2 y  sin 2 z 2
x y z

IMTSINSTITUTE.COM
MATHEMATICS-I 60

Solution

u = log(tanx + tany + tanz)


u 1
 .sec 2 x
x tan x  tan y  tan z
u sin 2 x.sec 2 x
sin 2 x 
x tan x  tan y  tan z
u 2 tan x
sin 2 x 
x tan x  tan y  tan z
u 2 tan y
Similarly, sin 2 y 
y tan x  tan y  tan z
u 2 tan z
sin 2 z 
z tan x  tan y  tan z
u u u 2(tan x  tan y  tan z )
Adding sin 2 x  sin 2 y  sin 2 z = =2
x y z tan x  tan y  tan z

Example 8

Verify Euler’s theorem for the following functions

y 1 3 3 2 2
(i) (ii) (iii) u = x +y +3x y-3xy
x x y2 2

Solution
y
(i) let f =
x
This is a homogeneous function of degree 0.
f f
We have to verify that x +y =0
x y
f y f y
 2 x 
x x x x
f 1 f y
 y 
y x y x
f f
Adding x +y =0
x y

Therefore Euler’s theorem is verified.

1 2 2 -1/2
(ii) let f = = (x +y )
x y
2 2

This is a homogeneous function of degree -1.


f f
We have to verify that x +y = -f.
x y

IMTSINSTITUTE.COM
MATHEMATICS-I 61

f
3
1 
  ( x 2  y 2 ) 2 .2 x
x 2
f
3

x   x2 ( x2  y2 ) 2
x
f
3
1 
  ( x 2  y 2 ) 2 .2 y
y 2
f
3

y   y2 ( x2  y2 ) 2
y
f f
3

Adding , x  y  ( x 2  y 2 )( x 2  y 2 ) 2
x y
1

 - (x  y ) 2 2 2
f
Therefore Euler’s theorem is verified.

3 3 2 2
(iii) u = x +y +3x y-3xy

u 2 2
=3x +6xy+3y
x

u 2 2
=3y +3x +6xy
y

u u
x y  x(3 x 2  6 xy  3 y 2 )  y (3 y 2  3 x 2  6 xy )
x y
3 2 2 3 2 2
= 3x +6x y+3xy +3y +6xy +3x
3 3 2 2
= 3x +3y +9x y+9xy
3 3 2 2
= 3(x +y +3x y+3xy )

= 3u

Therefore Euler’s theorem is verified.

Example 9

Verify Euler’s theorem in the following cases:

x2  y2 3 y x y
(i) u = sin (ii) u = x cos( ) (iii) u =
xy x x y
Solution
x2  y2
(i) Let u = sin
xy
-1 x2  y2
Sin u = =f
xy
This is a homogeneous function of degree 0.

IMTSINSTITUTE.COM
MATHEMATICS-I 62

f f
We have to verify that x +y =0.
x y
Differentiating with respect to x,
f xy.2 x  ( x 2  y 2 ). y x 2 y  y y ( x 2  y 2 )
  2 2 
dx ( xy ) 2 x y x2 y2
( x2  y2 )

x2 y
f x 2  y 2
x 
dx x y
f y2  x2
Similarly y 
dy xy
f f
Therefore x +y =0
x y
Therefore Euler’s theorem is verified.
3 y
(ii) u = x cos( )
x
u y y y
 3 x 2 cos( )  x 3 sin( ).( 2 )
x x x x
u y y
x  3 x 3 cos( )  x 2 y sin( ) ....(1)
x x x
u y 1
  x 3 sin( ).
y x x
u y
y   x 2 y sin( ) ....(2)
y x

Adding (1) and (2)

u u 3 y
x +y =3 x cos( ) =3u.
x y x

Therefore Euler’s theorem is verified.

2.5.1 SELF ASSESSMENT QUESTION – 5

x y u u
1. u= Verify Euler’s theorem x +y =0
x y x y
………………………………………………………………………………………………

………………………………………………………………………………………………

………………………………………………………………………………………………

x
1 2 f 2 f
2. If f  a tan   verify that 
 y xy yx
………………………………………………………………………………………………

………………………………………………………………………………………………

IMTSINSTITUTE.COM
MATHEMATICS-I 63

………………………………………………………………………………………………

2.6 Curvature and Radius of Curvature

The direction of the curve at a given point on the curve is determined by the tangent
at that point. The tangent line rotates as the point moves along the curve. The curvature of
the curve at a given point is defined as the rate of change of bending of the curve at the
point.

2.6.1 Cartesian Formula for Radius of curvature

Let P be any point on the curve y = f(x) and PT be the tangent making an angle
dy
 with x-axis. Then we know that  tan 
dx
Differentiating with respect to x,

d2y d
2
 sec 2 
dx dx
d  ds
= sec 2 
ds dx
ds sec  ds
2
  2 .
d d y dx
dx 2
(1+tan 2Ψ)secΨ dx
= [Since  cos Ψ]
d2 y ds
dx 2
(1+tan 2Ψ) 1+tan 2Ψ
=
d2 y
dx 2

IMTSINSTITUTE.COM
MATHEMATICS-I 64

dy 2 3 2
2
3 [1+( ) ]
(1+tan Ψ) 2
dx
= =
d2 y d2 y
dx 2 dx 2
3
(1+y12 ) 2
dy d2 y
i.e.,  = Where y1  and y2  2
y2 dx dx

Note: This formula does not hold good where the tangent at the point(x,y) is parallel to y
dy
axis. In that case is not defined. Since the value of  is independent of the choice of
dx
axis of coordinates, in this case we take the formula for  as

dy 2 3 2
[1+( ) ]
= dx
d2 y
dx 2
2.6.2 Parametric formula for Radius of Curvature

Let the parametric equations of the curve be x = f(t) and y = (t).

dy
dy y1
Then = dt  1
dx dx x
dt
d 2 y d dy
Also  ( )
dx 2 dx dx
d y1
= ( )
dx x1
d y1 dt
= ( )
dt x1 dx
x1 y11  y1 x11 1
= 2 . 1
x1 x
x y yx
1 11 1 11
= 3
x1
3 2 3
(1+y12 ) 2
y1 x1
3
=  [1  1 2 ] . 1 11 1 112
y2 x x y yx
3
( x1  y1 ) 2
2 2

= 1 11 1 11
x y yx

Example 1

3a 3 3
Find the radius of curvature at x = y = to the curve x +y = 3axy.
2

IMTSINSTITUTE.COM
MATHEMATICS-I 65

Solution
3 3
x +y = 3axy …(1)

Differentiating with respect to x

dy dy
2 2
3x +3y dx = 3a( x  y)
dx
dy 2
( y  ax)  ay  x 2
dx
dy ay  x 2
 ...(2)
dx y 2  ax
3a 2 9a 2

dy 3a 3a
( , )  22 4  1
dx 2 2 9 a 3a2

4 2
Also
dy dy
2 ( y 2  ax)(a  2 x)  (ay  x 2 )(2 y  a)
d y dx dx

dx 2 ( y  ax)
2 2

9a 2 3a 2 3a 2 9a 2
2 
( )(a  3a )  (  )(3a  a )
d y 3a 3a 4 2 2 4
( , ) 
dx 2 2 2 9a 2 3a 2 2
(  )
4 2
(-4a - 4a) 4 32
 2
 8a  2  
3a 3a 3a
( )
4

2 32 3
(1  y1 ) (1  1) 2 6 2a 3 2a
  3a   
y2  32 32 16
3 2a
 
16

Example 2
2 3 3
Find the radius of curvature of the curve xy =a -x at the point (a,0).

Solution
2 3 3
The equation of the curve is xy = a -x .

Differentiating with respect to x,


dy 2 2
2xy +y = -3x
dx
dy 2 2
2xy = -y -3x
dx

IMTSINSTITUTE.COM
MATHEMATICS-I 66

dy  y 2  3 x 2

dx 2 xy
dy
(a,0)  
dx
dx  2 xy
let us consider  2
dy y  3 x 2
dx
(a,0)  0
dy
dy dx
2 ( y 2  3 x 2 )(2 x  2 y )  2 xy (2 y  6 x )
d x dx dy

dy 2
( y  3x )
2 2 2

d 2x 3a 2 (2a  0)  0 2a 2
2
( a ,0)   2 2
 2 
dy (3a ) 3a 3a
dx 3
[1  ( ) 2 ] 2 3
dy (1  0) 2 3a
  
2
d x 2 2
2
dy 3a
3a
 
2
Example 3
3 3 2/3 2/3
Prove that the radius of a curvature at the point (acos θ, asin θ) on the curve x +y =
2/3
a is 3asin θcos θ

Solution

The parametric equations of the curve are


3 3
x = acos θ, y = asin θ
dx 2 dy 2
=-3acos θsin θ; =3asin θ cos θ
d d
dy 3a sin 2  cos
   tan 
dx 3a cos 2  sin 
d2y d  sec 2  1
  sec 2
  
dx 2
dx  3a cos  sin  3a cos  sin 
2 4

2 32
(1  y1 ) 3
  (1  tan 2  ) 2 .3a cos 4  sin 
y2
 sec3  .3a cos 4  sin   3a sin  cos
Example 4
2 a 2 (a  x) a
Show that the radius of the curvature at(a,0) on the curve y = is
x 2
Solution
2 a 2 (a  x)
y =
x

IMTSINSTITUTE.COM
MATHEMATICS-I 67

Differentiating with respect to x,

dy x( a )  a 2 (a  x)1  a 2 x  a 3  a 2 x a3
2y    
dx x2 x2 x2
dy a3

dx 2 yx 2
dy
( )( a ,0)  
dx

dx 2 yx 2 dy
Consider  3 ; ( )( a ,0)  0
dy a dx
d 2x 2 dx
2
  3 [ x 2  2 xy ]
dy a dy
d 2x 2 2
( )
2 ( a ,0)
  3 ( a 2  0)  
dy a a

dx 2 3 2
[1  ( ) ] 3
dy (1  0) 2 a
  
2
d x  2 2
2
dy a
a
 
2

Example 5

Find the radius of the curvature at the point ‘θ’ on the curve x = a(cos θ+ θsin θ),y = a(sin θ-
θcos θ).

Solution
x = a(cos θ+ θsin θ)

dx
= a(-sin θ+sin θ+ θcos θ) = a θcos θ
d
y = a(sin θ- θcos θ)
dy
= a(cos θ-cos θ+ θsin θ)=a θsin θ
d
dy dy d a sin 
   tan 
dx d dx a cos
d2y d sec 2  1
 sec 2
  
dx 2
dx a cos a cos3 
2 32
(1  y1 ) 3
  (1  tan 2  ) 2 .a cos3   sec3  .a cos3 
y2
 a

IMTSINSTITUTE.COM
MATHEMATICS-I 68

2.6.4 SELF ASSESSMENT – VI

1 x2  y 2  u u 1
1. If U  tan   show that x  y  sin 2u
 x y  x y 2

……………………………………………………………………………………………..

……………………………………………………………………………………………..

……………………………………………………………………………………………..

IMTSINSTITUTE.COM
MATHEMATICS-I 69

UNIT – III

3.0 Introduction
3.1 Objectives
3.2 Integration by Parts
3.2.1 SELF-ASSESSMENT QUESTION - I

3.3 Definite Integral

3.3.1 Definition of Definite Integral


3.3.2 Properties of Definite Integral
3.3.3 Self Assessment Question – I

3.4 Bernoulli’s Formula

3.4.1 Reduction Formula


3.4.1 Self Assessment Question – II

3.5 Particular Integral

3.5.1 Self Assessment Question – III

IMTSINSTITUTE.COM
MATHEMATICS-I 70

3.0 Introduction

In this chapter, we have to deal with Integration for Higher grade level and
understanding the formulae of the integration, Leibnitz Formulae and applied in to the
problem. Student study for the particular integrals and all the types of the integration.

3.1 Objectives

To develop the skill in the Integration

To create new idea to application oriented sum in the particular


integration.

To understanding the way of approach for the integration in


applied sciences.

3.2 Integration by Parts

d dv du
If u and v are functions of x we know that (uv) = u v
dx dx dx
Integrating both sides with respect to x, we get

d du du
 dx (uv) dx   u dx dx   v dx dx
i.e.  d(uv)   u dv   v du
uv =  u dv   v du
  u dv  uv   v du
This is called the by parts rule.

Note: By – parts is applied when the integral is a product of two functions. The evaluation of
this integral depends upon the proper choice of u and v. u is chosen such that  v du is
easily integrated.

Example 1

Integrate the following with respect to x.

x n 2
(i) x e (ii) x log x (iii) x sin 2x (iv) x sin x.

Solution

IMTSINSTITUTE.COM
MATHEMATICS-I 71

(i)  x e dx
x x x
Take u = x and dv = e dx. Then du= dx; V=e
  x e dx = uv -  v du
x

= x e -  e dx
x x

x x
= x e –e + c
x
= e (x-1) + c
(ii)  x log x dx
n n
Take u = log x and dv = x dx

1 x n 1
du = dx; v 
x n 1
  x log x dx = uv -  v du
n

x n 1 x n 1 1
= log x  dx
n 1 n 1 x
x n 1 1
=
n 1
logx 
n 1  x n dx
n 1
x x n 1
= logx  c
n 1 (n  1) 2
x n 1  1 
=  log x  c
n 1 n 1
(iii)  x sin 2x dx = Take u = x; dv = sin 2x dx
 cos 2x
du = dx ; v=
x
  x sin 2x dx = uv -  v du

  cos2x   cos2x
=x   dx
 2  2
x cos 2x sin 2x
=   c
2 4
(iv)  x sin x dx Take u = x;
2 2
dv = sin x dx
du = dx ; v =  sin x dx
2

1  cos 2 x
= dx
2
1 sin 2 x 
 x
2 
=
2
  x sin x dx = uv -  v du
2

1 sin 2 x  1  sin 2 x 
x x     x  dx
2 
=
2 2  2 
x 2 x sin 2 x x 2 cos 2 x
=    c
2 4 4 8
x 2 x sin 2 x cos 2 x
=   c
4 4 8

IMTSINSTITUTE.COM
MATHEMATICS-I 72

Example 2

Integrate the following

-1 -1 -1 x sin -1 x
(i) sin x (ii) tan x (iii) x tan x (iv)
1- x 2
Solution

(i)  sin x dx
-1 -1
Take u = sin x dv = dx
1
du = dx, v  x
1- x 2
 sin x dx = uv -  v du
-1

x
= x sin x- 
-1
dx
1 x2
 -2x dx = dt
2
Put 1 - x = t
d t/2
  sin x dx = x sin x + 
-1 -1

t
1
1 t 2
-1
= x sin x + c
2 1/2
-1
= x sin x + 1 x2  c

 tan x dx
-1 -1
(ii) Take u =tan x; dv = dx
1
du  dx; vx
1 x2
  tan x dx = uv - v du
-1

-1 x
= (tan x) x- dx
1 x 2
1
= x tan 1 x  log(1  x 2 )  c
2

 x tan
-1
(iii) x dx

Take u  tan -1 x; dv  x dx
1 x2
du  dx ; v 
1 x2 2
  x tan -1 x dx  uv -  v du

x2
tan 1 x   x2
2
1
=
1 x 2
dx
2

IMTSINSTITUTE.COM
MATHEMATICS-I 73

x2 1 x2
= tan x  
1
dx
2 2 1 x2
x2 1 x 2 11
= tan 1x   dx,
2 2 1 x2

x 2 tan 1 x 1  1 
=   1 -  dx
2 2  1 x2 
x 2 tan 1 x 1
=   [x  tan 1x]  c
2 2

x sin 1 x dx
(iv)  1 x2

-1 x
Take u = sin x ; dv = dx
1 x2
1 x
du = ; v dx
1 x2 1 x2
2
put u = 1-x
du = -2x dx
du / 2
v =  u
=- 1 x2

x sin 1x dx
  1 x2
= uv-  v du

1
=  1  x 2 sin 1 x   1  x 2 dx
1 x2

=  1  x 2 sin -1 x  x  c

3.2.1 Self-Assessment Question - I


4

 2sin
2
1. Evaluate x sin 2 xdx
0

……………………………………………………………………………………………………….…
……..
……………………………………………………………………………………………………….…
……..

Ans: ¼

IMTSINSTITUTE.COM
MATHEMATICS-I 74

3.3 Definite Integral


3.3.1 Definition of a Definite Integral

Let f be a function defined on a closed integral [a,b]. Let P be a partition of [a, b]

with partition points x0, x1,…xn where ax0,<x1,<x2 <……<xn=b. Choose points xi* in [xi-1, xi]

and let xi  xi  xi 1 and || p || max{xi } . Then the definite integral of the form

b n

 f ( x)dx  lim  f (x )xi , if this limit exists. If this limit does exist then it is called
*
i
|| p||0
a i 1

integrable on the interval [a, b].

The definite integral of f (x) between the limits x=a and x= b is defined by

 f (x) dx and its value is F (b) – F (a).


a

Here ‘a’ is called the lower limit and ‘b’ is called the upper limit of the integral, and F
(x) is the integral of f (x).

The value of the definite integral is obtained by finding out the indefinite integral first

and then substituting the upper limit and lower limit for the variable in the indefinite integral.

3.3.2 Properties of Definite Integral

Let  f (x) dx  F (x)  C.


b

 f (x) dx  F (b)  F (a)  [F (x)]


b
Then a
a

Property 1

b b

 f (x) dx   f (t) dt .
a a
Proof

 f (x) dx F(x)  F (b)  F (a) .


b
a
a
b

 f (t) dt F(t)  F (b)  F (a) .


b
a
a
b b
  f (x) dx   f (t) dt .
a a

IMTSINSTITUTE.COM
MATHEMATICS-I 75

Property 2:

b b

 f (x) dx   f (x) dx .
a a

Proof :

a
=  f (x) dx  -F(x)
a
RHS b
b

=  [F(a)  F(b)]
= [F(b)  F(a)]
b
=  f (x) dx
a

Property 3 :
b c b

 f(x)dx   f(x) dx   f(x) dx


a a c
Proof
c b
RHS   f(x) dx   f(x) dx
a c

 f(x)  [F(x)]cb
c
a
= F(c) - F(a) + F(b) – F(c).
= F(b) – F(a).
b
  f(x)dx
a
Property 4 :

a a

 f (x) dx   f (a - x) dx .
0 0
Proof:

Put a - x =t
dx = -dt
When x = 0, t= a; When x= a, t= 0.

0 a
R.H.S = -  f (t) dt   f (t) dt
a 0
a
=  f (x) dx by property (1)
0
a a
  f (x) dx   f (a - x) dx
0 0

Property 5:

IMTSINSTITUTE.COM
MATHEMATICS-I 76

2a a a

 f (x) dx   f (x) dx   f (2a - x) dx


0 0 0

Proof
2a a 2a

 f (x) dx   f (x) dx   f (x)dx .


0 0 a
…… (1)

nd
Put x = 2a – t in the 2 Integral.
dx = -dt
When x= a, t=a; x=2a, t=0.

2a 0
  f (x) dx    f (2a  t) dt
0 a
a
=  f (2a - t) dt
0
a
=  f (2a - x) dx
0
Using this in equation (1) we get

2a a a

 f (x) dx   f (x) dx   f (2a - x) dx


0 0 0

Property 6:

2a 2a
i) If f (2a-x) = f (x) then  f (x) dx  2  f (x) dx
0 0
2a
ii) If f (2a-x) = -f (x) then  f (x) dx =0.
0

2a a a
i) by property 5  f (x) dx   f (x) dx   f (2a - x) dx .
0 0 0
a a
=  f (x) dx   f (x) dx .
0 0
a
= 2  f (x) dx .
0
ii) If f (2a-x) = -f (x) then

2a a a

 f (x) dx   f (x) dx   f (2a - x) dx


0 0 0
a a
=  f (x) dx   f (x) dx.  0 .
0 0
Property 7

IMTSINSTITUTE.COM
MATHEMATICS-I 77

a a
(i)  f (x) dx  2 f (x) dx if f (x) is an even function.
a 0
a
(ii)  f (x) dx =0 if f (x) is odd function.
a

Proof:
a a a

 f (x) dx   f (x) dx   f (x) dx


a a 0
0 a
=
a
 f (-x) dx   f (x) dx0
a a
=  f (t) dt   f (x) dx
0 0
st
(Putting –x = t in the 1 integral)
a a
=  f (x) dx   f (x) dx
0 0
a
= 2  f (x) dx
0
Suppose f (x) is an odd function,
Then f (x) = - f (-x).
a 0 a

 f (x) dx   f (x) dx   f (x) dx


a a 0
0 a
=
a
 - f (-x) dx   f (x) dx 0
(Putting –x = t in the integral)
0 a
=  - f (t) dt   f (x) dx .
a 0
a a
=   f (x) dx   f (x) dx .
0 0
=0

Property 8

b b

 f (x) dx   f (a  b - x) dx .
a a

Proof:
b
RHS =  f (a  b - x) dx
a
Put a + b – x = t.
 dx = -dt.
When x = a, t=b, x=b, t=a.

IMTSINSTITUTE.COM
MATHEMATICS-I 78

a b
RHS=  f (t) (-dt)   f (t) dt
b a
b
=  f (x) dx  L.H.S .
a

Example : 1

π
2

 sin
2
i) Evaluate x dx.
0

Solution

π
2

 sin
2
Let I= x dx.
0
π
π 
2

 sin  2  x  dx
2
= (by property 4)
0
π
2

 cos
2
= x dx (2)
0
Adding (1) and (2)
π π
2 2

 (sin x  cos ) dx   dx
2 2
2I =
0 0

= x  0  π
π
2

2

I=
4

Example 2

 x sin x dx
2
Evaluate
0
Solution

 x sin
2
Let I = x dx (1)
0
π

 (π - x) sin
2
= (π - x) dx
0
π

 (π - x) sin
2
= x dx (2)
0
Adding (1) and (2)

IMTSINSTITUTE.COM
MATHEMATICS-I 79

 [x sin x  (π - x)sin 2 x] dx
2
2I =
0
π

 π sin
2
= x dx .
0

 1 - cos 2x 
π
=  π 
0
2
 dx

π
π sin2x 
= x 
2 2  0
2
=
π
π   π
2 2
2
π
I=
4
Example 3
π
2
sinx
Evaluate 
0 sinx  cosx
dx

Solution

π
2
sinx
Let I = 
0 sinx  cosx
dx . …… (1)

π 
π sin x   x 
2 
2

=  π  π 
dx .
0
sinx   x   cosx  x 
2  2 
π
2
cos x
= 
0 sin x  cos x
dx ….. (2)

Adding (1) and (2)


π
2
sin x  cos x
2I = 
0 sin x  cos x
dx
π

 dx  x 
2
π

π
2
= 0
0
2

I=
4
Example 4

π
4

Evaluate =  log(1  tan  ) d


0
Solution
π
4

Let I =  log (1  tan ) d


0

IMTSINSTITUTE.COM
MATHEMATICS-I 80

π
4
  
=  log 1  tan 4    d
0

  
π

4 tan  tan  
=  log 1  4
 d

0  1  tan tan  
 4 
π
4
 1  tan  
=  log 1  1  tan   d
0
π
4
1  tan   1  tan  
=  log 
0
1  tan   d

π
4
 2 
=  log 1  tan   d
0
π

 log 2  log(1  tan  )d


4

=
0
π 
4 4

=  log 2d   log (1  tan  ) d


0 0

4
= log 2 [] 0 -I

2I = log 2 π I 
π
log 2
4 8
Example 5

π
x sin x dx
Evaluate  1  cos
0
2
x
Solution

π
x sin x
Let I =  1  cos
0
2
x
.... (1)

π
(π - x) sin (π - x)
= 
0
1  cos 2
(x - π)
dx

π
(π - x) sin
=  1  cos
0
2
x
dx .... (2)

Adding (1) and (2)


x sin x  (π - x)sin x
π
2I = 
0
1  cos 2 x
dx

 sin x
π
=  1  cos
0
2
x
dx

Put cos x = t, - sin xdx = dt


When x= 0, t= cos 0 = 1
When x=, t= cos  = -1

IMTSINSTITUTE.COM
MATHEMATICS-I 81


-1
(ie) 2I =  1 t
1
2
dt
1 1
dt dt
= π  2π 
-1
1 t 2
0
1 t2
= 
2π tan 1t 0  2π tan 11  1

 
= 2π 
4
2
I =
4

Example 6
π
x tan x
Evaluate  sec x  tan x dx
0
Solution
π
x tan x
Let I =  sec x  tan x dx
0
. . . .(1)

(π  x) tan (π  x)
π
=  sec (x - π)  tan (π - x) dx
0

- (π  x) tan x
π
=  - sec x  tan x dx
0

(π  x) tan x
π
2I =  sec x  tan x dx
0
Adding (1) and (2)

π
π tan x
=  sec x  tan π dx
0

x sin x
π
=  cos x dx
1 sin x
0 
cos x cos x
π
π sin x
=  dx
0
1  sin x
π
π sin x(1 - sinx)
= 
0
1 - sin 2 x
dx

π
π (sin - sin 2 x)
= 0 cos 2 x dx
π
= π  (secx tan x - tan 2 x) dx
0

IMTSINSTITUTE.COM
MATHEMATICS-I 82

π
= π  (sec x tan x - sec 2 x  1) dx
0

πsec x - tan x  x 0

=
= π sec π - tan x  π)  sec 0  tan 0  0 
= π - 1 - 0  π   1  0 
=  (-2)
ππ  2 
I =
2
Example 7

 x (1 - x)
10
Evaluate dx
0

Solution

 x (1 - x)
10
dx
0
1

 (1 - x)1 - (1 - x)
10
= dx
0
1 1

 (1 - x)x dx   (x10  x11 ) dx


10
=
0 0
1
 x11 x12  1 1
=     
 11 12  0 11 12
12  11 1
= 
132 132

IMTSINSTITUTE.COM
MATHEMATICS-I 83

Example 8

log (1  x)
1
Evaluate = 
0
1 x2
dx

log (1  x) dx
1
Let I = 
0
1 x2
Put x = tan   dx = sec  d
2

When x = 0,  =0
π
When x = 1,  =
4
log (1  tan θ)
π/4
I = 0
1  tan θ
2
sec 2 θ d θ

log (1  tan θ)
π/4
= 0
sec 2
sec 2 θ d θ

π/4
=  log (1  tan θ) d θ
0

log (2) see Example 5


π
=
2

Example 9

π -β
θdθ β π
Show that  sin θ  π log cot 2, 0  β  2
β
Solution :

π -β
θdθ
I =  sin θ
β
..... (1)

β  π -β -θ
π -β

=  sin (β  π  β  θ) d θ
β
(using property (6))

π -β
π-θ
= β
sin (π - θ)
π -β
π-θ
= β
sin θ
dθ . . . . . . (2)

π -β
π
Adding (1) and (2), 2I = 
β
sin θ

π β
 θ
= π log tan 
 2 β

IMTSINSTITUTE.COM
MATHEMATICS-I 84

 π β β
= π log tan     log tan 
  2 2 2
 β β
= π log cot  log cot 
 2 2
β β
2I = 2π log cot  I  π log cot
2 2
Example 10

b b
Using property  f (x) dx   f (a  b  x) dx
a a
3

 (x - 20 x 3  150 x 2  500x  625) dx


4
Evaluate
2
Solution

 (x - 20 x 3  150 x 2  500x  625) dx


4

2
3

 (x - 4c1x 3 5  4c 2 x 2 52  4c 3 x 3 53  54 ) dx
4
=
2
3 3

 (x - 5) dx   (5  x) dx
4 4
=
2 2
3

 (2  3 - x)
4
= dx
2
By the given property

3
3
 x5 
=  x dx   
4

2  5 2
35 25 211
=  
5 5 5
Example 11
 /2
Show that   log(tan x  cot x)dx   log2
0
Solution
 /2
LHS   log(tan x  cot x)dx
0
 /2
 sin x cos x 
  log  cos x  sin x  dx
0
 /2
 1 
  log  sin x cos x  dx
0
 /2  /2
   log sin x dx    log cos x dx.
0 0

IMTSINSTITUTE.COM
MATHEMATICS-I 85

 
 log 2  log 2   log 2.
2 2
Example 12
 /4

Show that  (log sin x  cos x)dx   4 log 2

4
Solution
 /4
  log(sin x  cos x)dx
 / 4
 /4
 1 1 
 
 / 4
log 2 
 2
sin x 
2
cos x  dx

 /4
 
 
 / 4
log 2 sin   x  dx
4 
 /4  /4
 
  log 2 dx 
 / 4

 / 4
log sin   x  dx
4 
 /4  /4
1  
 log 2  dx   log sin  x   dx
2  / 4  / 4  4

Put xt
4
dx= dt

When x , t  0.
4
 
When x   , t 
4 2
 /2
1   
 RHS  log 2.     log sin t dt
2 4 4 0
 
 log 2.  log 2.
4 2

 log 2.
4
Example 13
a a
a
Prove that  x f ( x)dx 
0
2 0
x f ( x)dx. If f(a-x)  f(x)

Solution
a a
LHS   xf ( x)dx   (a  x) f (a  x)dx
0 0
a a

 af (a  x)dx   xf (a  x)dx
0 0
a a a
(ie)  xf ( x)dx   af ( x)dx   xf ( x)dx
0 0 0
 [ f (a  x)  f ( x)]

IMTSINSTITUTE.COM
MATHEMATICS-I 86

a a
 2  xf ( x)dx  a  f ( x)dx
0 0
a a
a
  xf ( x)dx 
2 0
(ie) f ( x)dx
0

Example 14
b b
If f (a  b  x)  f ( x) prove that 2  xf ( x)dx  f (a  b)  f ( x)dx
a a
Solution

Put a + b – x = t
dx = - dt
When x= a, t=b.
When x=b, t=a.
b b
  xf ( x)dx    (a  b  t ) f (a  b  t )dt
a a
b
  (a  b  t ) f (a  b  t )dt
a
b b
 (a  b)  (a  b  t )dt   tf (a  b  t )dt
a a
b b
 (a  b)  (a  b  x)dx   xf (a  b  x)dx
a a

b b
 (a  b)  f ( x)dx   xf ( x)dx
a a

 f (a  b  x)  f ( x)

b b
 2  xf ( x)dx  (a  b)  f ( x)dx
a a

Example 15
2a
If f (a  x)   f (a  x)prove that  f ( x)dx  0
0
Solution
2a a 2a

 f ( x)dx   f ( x)dx   f ( x)dx


0 0 0
....(1)
nd
Consider the 2 integral
Put t=x-a.
dt=dx.
When x=a, t=0.
x=2a, t= a.

IMTSINSTITUTE.COM
MATHEMATICS-I 87

2a a
  f ( x)dx   f (a  t )dt
0 0
a
   f (a  t )dt
0

 f (a  x)   f (a  x)
(Property 4)
a
   f (t )dt
0
a
   f ( x)dx
0
2a a a
 From (1)  f ( x)dx   f ( x)dx   f ( x)dx 0
0 0 0
Example 16
1

 cot
1
Evaluate (1  x  x 2 )dx
0
Solution
1 1
1
 cot (1  x  x )dx   tan
1 2 1
dx
0 0
1 x  x2
1
1
  tan 1 dx
0
1  x(1  x)
x 1 x
1
  tan 1 dx
0
1  x(1  x)
(property 4)
1 1
  tan xdx   tan 1 xdx
1

0 0
(using property 5)
1
 2  tan 1 xdx
0

 2x tan x   
1
x 1 1
dx
1 x
0 2
0
1
 1 
 2  x tan 1 x  log(1  x 2 )
 2 0
 1 
 2   log 2
4 2 

  log 2
2
3.3.3 Self Assessment Question – I

IMTSINSTITUTE.COM
MATHEMATICS-I 88

dx
1. Evaluate  x( x 4
 1)

……………………………………………………………………………………………………….…
……..
……………………………………………………………………………………………………….…
……..

1  x4 
Ans: log  4   c
4  x 1

1 x
2. Evaluate  1 x
dx

……………………………………………………………………………………………………….…
……..
……………………………………………………………………………………………………….…
……..
Ans: Sin 1 x  1  x 2  c

3.4 Bernoulli’s formula

x
3
Consider the integral a sin 2 xdx . We can evaluate this integral by
repeatedly applying ‘by parts’ rule.
3
Let u=x dv = sin 2x dx
cos 2 x
du  3 x 3 dx v
2
 I   x 3 sin 2 xdx
= uv   vdu
cos 2 x cos 2 x 2
  x3  .3 x dx
2 2
cos 2 x
Again take u = 3x
2
dv   dx
2
sin 2 x
du = 6xdx v
4

Again applying ‘by parts’ rule we get

 cos 2 x   2  sin 2 x   sin 2 x 


4    4 
I  x3    3 x      6 xdx
 2   
 cos 2 x  2  sin 2 x   sin 2 x 
4    4 
 x3    3 x      6 xdx
 2  

IMTSINSTITUTE.COM
MATHEMATICS-I 89

sin 2 x
Again take u = 6x dv   dx
4
cos 2 x
du = 6dx v
8

 cos2x  2 sin2x   cos2x  cos2x


I  x3    3x     6 x  .6dx
 2   4   8  8

 cos 2 x  2 sin 2 x   cos 2 x   sin 2 x 


 x3     3x     6 x   6  ..........(1)
 2   4   8   16 

If u’, u”, u’“, etc denote first, second, third derivatives of the function u and v1, v2, v3 etc are
the successive integrals of functions of the function v then form (1), we arrive at a formula
for integration of u dv.

(i.e.)  udv  uv  u ' v 1  u" v2  u ' " v3  .......

This is called Bernoulli’s formula for integration.

Example 1

ax
Evaluate e .x 3 dx

Solution

Here u  x3 dv  e ax dx
e ax
u '  3x 2 v
a
e ax
u" 6 x v1  2
a
e ax
u'"  6 v2  3
a
e ax
v3  4
a

  e ax .x 3 dx  uv  u ' v1  u" v2  u ' " v3


e ax  e ax   e ax   e ax 
 x3  3 x 2  2   6 x 3   6 4   4
a a  a  a 
 x 2 3x 2 6 x 6 
e   2  3  4c
ax

a a a a 
e ax

 4 a 3 x 3  3a 2 x 2  6ax  6  c
a

IMTSINSTITUTE.COM
MATHEMATICS-I 90

Example 2

3
 x e dx
x
Evaluate
Solution

3
 x e dx
x

u  x3 dv  e x
u '  3x 2 v  ex
u" 6 x v1  e x
u'"  6 v2  e x
v3  e x
  x 3e x dx  x 3e x  3 x 2 e x  6 xe x  6e x  c

 e3 x 3  3x 2  6 x  6  c 
Example 3

2
 x e dx
2 x
Evaluate

Solution

2
 x e dx
2 x

 e 2 x   e 2 x   e 2 x 
 x2    2 x    2 c
 2   4   8 
2 x  x x 1

 e      c
 2 2 4


4

e 2 x
2x  2x 1  c 
3.4.1 Reduction formula

Example 1

x
m
1. Reduction formula for (log x) n dx
Let I m ,n   x m (log x) n dx
Let u  (log x) n and dv  x m dx
(log x) n 1 x m 1
du  n dx and v 
x m 1
Then applying by parts Rule,

 n x
m 1
 x m 1 (log x) n 1
m  1  m  1
I m,n  (log x)  .n dx
 x

IMTSINSTITUTE.COM
MATHEMATICS-I 91

x m 1 n

m 1
(logx) n 
m 1  x m (logx) n 1 dx
m 1
x n
(i.e) I m , n  (log x) n  I m ,n 1
m 1 m 1
Example 2

Reduction formula for I n   x n e ax dx

Solution

Let u  xn dv  e ax dx
e ax
du  nx n 1dx v
a
x n e ax e ax n 1
In   .nx dx
a a
x n e ax n n 1 ax
   n e dx
a a
n ax
x e n
 In   I n 1
a a
Example 3

 sin
n
Reduction formula for xdx

Solution

Let I n   sin n xdx


  sin n 1 x sin xdx
u  sin n 1 x dv = sinx
n-2
du = (n-1) sin x cosx dx v=-cos x
 I n  sin n 1x(cosx)   (n  1)sin n  2 xcosx.(cosx)dx
=  sin n 1xcosx  (n  1)  sin n  2 xcos 2 x.dx
 sin n 1xcosx  (n  1)  sin n  2 x(1  sin 2 x)dx
 sin n 1xcosx  (n  1)  sin n  2 xdx  (n  1)  sin n x dx
 sin n 1xcosx  (n  1)I n  2  (n  1)I n

 (1  n  1)I n  sin n 1xcosx  (n  1)I n  2


sin n 1xcosx n  1
In    I n 2
n n

IMTSINSTITUTE.COM
MATHEMATICS-I 92

Example 5

π 2

 sin
n
Reduction formula for x dx
0

Solution

π 2

Let I n   sin n x dx
0
π 2

 sin
n -1
= x (sin x) dx
0
n-1
Let = u=sin x v=sinx dx
n-2
du = (n-1)sin x cos x dx dv= -cos x
π

 π 2
In = I n   cosxsin n 1x 0 2   cosx(n  1)sin n  2 x cosx dx
0
π
2
 0  (n  1)  sin n  2 xcos 2 x dx
0
π
π 2 2
= (n  1)  sin n  2 xdx  (n  1)  sin n x dx
0 0

 I n  (n  1)I n  2  (n  1)I n
 (1  n  1)I n  (n  1)I n  2
(n  1)
In  I n 2
n
Evaluation of In

Case 1: let n be an even integer


n -1
In  I n2
n
(n  1) n - 3
 . I n 4
n n-2
Proceeding in this way
(n  1) n - 3 n  5 1
In  . . ........ .I 0
n n-2 n4 2

2

But I 0   dx 
0
2
(n  1) n  3 n  5 1
In  . . ..... .
n n2 n4 22
Case 2 : let n be an odd integer.
(n  1) n  3 2
Then In  .  I1
n n2 3

IMTSINSTITUTE.COM
MATHEMATICS-I 93

I1   sin xdx   cos0 2  1


1

But
0

(n  1) n  3 n  5 2
 In  . 
n n2 n4 3
Example 6

π
2

 cos x dx
n
Reduction formula for
0

Solution

π
2
Let I n   cos n x dx
0
π
2

 cos
n -1
= x cos x dx
0
n-1
Let u = cos x dv = cos x dx
n-2
du = (n-1) cos x(-sin x dx) v= sin x

 cos xsinx 
π 2
  sinx(n  1)cos n  2 xsin x dx
2
n 1
 In 0
0
π
2
 0  (n  1)  cos n  2 x(1  cos 2 x)dx
0
= (n-1) In-2 – (n-1) In
In +(n-1) In = (n-1)In-2
(1+n-1) In = (n-1)In-2

n 1
 In  I n 2
n
Proceeding as in the last example

n 1 n  3 n  5 1 π
When n is even . . In 
...... .
n n2 n4 2 2
n 1 n  3 n  5 2
When n is odd I n  . . ......
n n2 n4 3
Example 8


4
1
If I n   tan n xdx prove that I n  I n  2  and hence evaluate I5
0
n 1

Solution

IMTSINSTITUTE.COM
MATHEMATICS-I 94


4
I n   tan n xdx
0

4
  tan n  2 x(sec 2 x  1)dx
0
 
4 4
  tan n  2 xd (tan x)   tan n  2 xdx
0 0

 tan n 1 x  4
In     I n2
 n 1 0

1
 I n  I n2 
n 1
1
 In   I n2
n 1
1
I5   I3
4
1
I 3   I1
2

11 
 I 5    I1 
42 
1 1
   I1
4 2

1 4
    tanxdx
4 0

log sec x0 4


1 

4
1
  log 2
4
1
 log 2
4

Example 9

e
x
Reduction formula for x n dx
0
Solution

Let I n   e  x x n dx
0


  ex x n    nx

0
n 1  x
e dx
0

IMTSINSTITUTE.COM
MATHEMATICS-I 95


 0  n  x n 1e  x dx
0

 I n  nI n 1
Also  I n 1  ( n  1) I n  2
I n  2  (n  2) I n 3
Multiplying all these results
I n  n(n  1)(n  2).......2.1.I 0

 
I 0   e  x dx  e  x

0 1
0

 I n  n!

3.4.1 Self Assessment Question – II

 cos
n
1. Reduction formula for xdx
……………………………………………………………………………………………..

……………………………………………………………………………………………..
cos n 1 x sin x n  1
Ans: In   I n2
n n

2

 sin
7
2. Evaluate : xcox5 xdx
0
……………………………………………………………………………………………..

……………………………………………………………………………………………..
1
Ans:
120

IMTSINSTITUTE.COM
MATHEMATICS-I 96

3.5 Particular Integral

Example 1
2 3 2x
Solve (D -4D+3)y=x e
Solution
2
The auxiliary equation is m -4m+3=0
(m-1) (m-3)=0
m=1, 3
x 3x
The complementary function is y=Ae + Be
1
PI  x 3e 2 x
D  4D  3
2

1
 e2 x x3
( D  2)  4( D  2)  3
2

1
 e2 x 2 x3
D  4D  4  4D  8  3
1
 e2 x 2 x3
D 1
1
 e2 x x3
1 D 2

 e (1  D 2 ) 1 x 3
2x

 e 2 x (1  D 2 ) x 3
 e 2 x ( x 3  6 x )
The complete solution is y  Ae  Be  e ( x  6 x)
x 3x 2x 3

IMTSINSTITUTE.COM
MATHEMATICS-I 97

Example 2

d2y dy
Solve  5  6 y  e x cos 2 x
dx 2 dx
Solution
2
The auxiliary equation is m -5m+6=0
(m-2) (m-3)=0
m=2, 3
x 3x
The complementary function is y=Ae + Be
1
PI  .e x cos 2 x
D  5D  6
2

1
 ex cos 2 x
( D  1)  5( D  1)  6
2

1
 ex 2 cos 2 x
D  3D  2
1
 ex cos 2 x
 4  3D  2
1
 e x cos 2 x
3D  2
3D  2
 e x cos 2 x
 40
ex
 (6 sin 2 x  2 cos 2 x)
40
 e x (3 sin 2 x  cos 2 x)

20
 The complete solution is
ex
y  Ae 2 x  Be3 x  (3 sin 2 x  cos 2 x)
20
Example 3
Solve ( D  2 D  1) y  x sin x
2 ex

Solution
2
The auxiliary equation is m -2m+1=0
m=1, 1
 The complementary function is y=Ae + Be
x 3x

1
PI  xe x sin x
D  2D  1
2

1
 xe x sin x
( D  1) 2

1
 ex x sin x
( D  1  1) 2
1
 e x 2 x sin x
D
 2D  1
 e x  x  2  2 sin x
 D D
 2
 e x  x   ( sin x)
 D

IMTSINSTITUTE.COM
MATHEMATICS-I 98

 e x ( x sin x  2 cos x)
 The complete solution is
y  e x ( Ax  B)  e x ( x sin x  2 cos x)

Example 4

Solve ( D 2  D  2) y  e 2 x  e x

Solution
2
The auxiliary equation is m – m – 2 = 0
 m=2, -1
 The complementary function is y=Ae + Be
x 3x

1 1
PI  e2 x  2 ex
D D2
2
D D2
1 1
 e2 x  ex
( D  2)( D  1) 11 2
1 1 1
 . e2 x  e x
3 D2 2
1 2x 1 x
 xe  e
3 2
 The complete solution is
1 1
y  Ae 2 x  Be x  xe 2 x  e x
3 2
Example 5

Solve ( D 2  6 D  9) y  e 3 x
Solution
2
The auxiliary equation is m –6 m+9 = 0
(m-3) =0  m=3, 3.
2

 The complementary function is


y  e3 x ( Ax  B)
1
PI  2 e3 x (failure case)
( D  6 D  9)
1
 e3 x
( D  3) 2

x2 3x
 e
2
x2 3x
 the complete solution is y  e ( Ax  B ) 
3x
e
2

3.5.1 Self Assessment Question - III

1. Solve : D 2  2 D  3) y  e 2 x (1  x 2 )

………………………………………………………………………………………….

IMTSINSTITUTE.COM
MATHEMATICS-I 99

………………………………………………………………………………………….
3 x e 2 x 2 12 x 87
Ans: y  Ae  Be 
x
(x   )
5 5 25

2. Solve : D  4 D  4) y  8 x e Sin 2 x
2 2 2x

………………………………………………………………………………………….
………………………………………………………………………………………….
 3 
Ans: y  ( A  Bx)e 2 x  2e 2 x  x 2   sin 2 x  2 x cos 2 x 
 2 

IMTSINSTITUTE.COM
MATHEMATICS-I 100

UNIT – IV

4.0 Introduction
4.1 Objectives
4.2 Non-linear differential equation of first order standard type
4.2.1 Complete solution
4.1.2 Singular Solution
4.1.3 General solution
4.1.4 Charpits Method
4.1.5 Standard type I
4.1.6 Standard type II
4.1.7 Standard type III
4.1.8 Standard type I
4.1.9 Self Assessment Question - I

4.2 Lagrange’s Linear Partial Differential Equations


4.2.1 Self Assessment Question - II

IMTSINSTITUTE.COM
MATHEMATICS-I 101

4.0 Introduction

In this chapter, we have to deal with Partial Differential Equation which involves
higher powers in non-linear partial differential equation, as understanding the concept of
differential equation. In the differential equation with standard type are deal with all features
in the high level and low level magnitude of the equations. Students will study for the
second order differential equation with all standard types.

4.1 Objectives

To develop the skill in the differential equation with standard types.

To create new idea to application oriented in the all applied sciences, like tsunami
tides which deal with high and low attitude of the waves.

To understanding the way of approach for the non-linear differential equation in all
applied sciences.

4.2 Non-linear differential equation of first order standard type

z
A partial differential equation involving the first order partial derivatives ( p)
x

z
and ( q ) is called a Linear partial differential equations. Partial differential equations
y

which involve the higher powers of p, q, pq are called non-linear partial differential

equations. Linear partial differential equations are obtained by eliminating arbitrary

constants or arbitrary functions.

4.2.1 Complete solution

A solution of a partial differential equation which contains as many arbitrary

constants as the number of independent variables is called a complete solution. The

complete solution is also referred to as complete integral. For example the partial differential

equation

f(x, y, z, p, q) = 0. (1)

may be obtained from the equation g(x, y, a, b) = 0. (2)

IMTSINSTITUTE.COM
MATHEMATICS-I 102

In this case (2) is called the complete solution of the partial differential equation (1).

There are two independent variables. In the solution (2). There are two arbitrary constants.

3.1.2 Singular Solution

The complete solution represents two parameters family of surfaces which may or

may not have an envelope.

The envelope if it exists, is obtained by eliminating ‘a’ and ‘b’ from

g ( x, y , z , a , b )  0 (1).


g ( x, y, z , a, b)  0, (2).
a


g ( x , y , z , a , b )  0. (3).
b

If the eliminent  (x, y, z)= 0 exists it is called the singular solution of equation (1).

Hence if f(x, y, z, p, q) = 0 is the partial differential equation whose complete solution is g(x,

y, z, a, b)=0 then the eliminent (if it exits) of

g ( x, y , z , a , b )  0 . (1).


g ( x , y , z , a , b )  0. (2).
a


g ( x , y , z , a , b )  0. (3).
b

Is called the singular solution or singular integral. The singular solution represents

the envelope of family of surface given by the general solution g x, y z , a, b   0. The

envelope in general may or may not exist.

4.1.3 General solution

IMTSINSTITUTE.COM
MATHEMATICS-I 103

In the non-linear partial different equation f(x,y,z,a,b)=0, if g(x,y,z,a,b)=0 is the

complete solution it may be possible for the existence of a relationship between the

arbitrary constants a and b. Suppose this relationship given by b=(a). Then the complete

solution takes the form  


g x, y , z , a,  , ( a )  0. This is a one parameter family of

surfaces of the partial differential equation f(x,y,z,p,q) = 0. If the family has an envelope it is

obtained by eliminating a from

g ( x, y, z , a,  (b))  0. (1)


and g ( x, y, z , a,  (b))  0. (2)
a

If this eliminent if it exists if called the general solution of equation (1). This solution

is also referred to as general integral.

Standard type I: f(p,q) = 0

Here the partial differential equation involves only p and q and the variables x, y, z are

absent.

Standard type II: z = px + qy + f (p, q)

This called the clairaut’s form

Standard type III: F (z, p, q) = 0.

Here the partial differential equation will contain p, q and the dependent variable z

only. The independent variables x and y are absent.

Standard type IV: Partial differential equation of the form f1(x, p) = f2(y, q). Here z is

absent and also it is in a separable form (i.e) the terms containing x and p only on one side

and terms containing y and q only on the other side.

4.1.4 Charpits Method

This is a general method of solving linear partial differential equation where the

given partial differential equation cannot be reduce to any one of the four general forms.

4.1.5 Standard type I;

IMTSINSTITUTE.COM
MATHEMATICS-I 104

F (p, q) = 0. (1)
Consider the equation
z = ax + by + c ………(2)
where a and b are connected by the relation f(a, b) = 0.

Differentiating the above equation partially with respect to x and y we get p = a, q =b.
Substituting these in the given partial differential equation w get f(a, b) = 0.
From the relationship f(a, b) =0 we can find b in terms of a, say b =  (a)
Substituting this in (2) we get, z = ax+  (a) y +c …….(3)
We see that (3) is the complete solution of equation (1).
Hence we note that for the linear partial differential equation of type (1), the
compete solution is of the form
z=ax + by + c where a and b are connected by relation  (a, b) = 0 or b=
g (a).
Here a and b are arbitrary constants.
To find the singular solution we have to determine the eliminent of a, and c from
z = ax + g (a) y + c
0 = x + g’ (a) y (differentiating partially with respect to a)
Differentiating partially with respect to c, 0 =1.
As the last equation in inconsistent no singular solutions will exist for the partial
differential equation of the type F (p, q) = 0.
Take c =  (a).
The complete solution becomes one parameter family
z = ax + g (a) y +  (a)
Differentiating partially with respect to a,
0 = a + g’ (a) + ’ (a)
Eliminating ‘a’ from the last two equations we get the general solution which is the
envelop of the single parameter family given by 1.

Example : 1

2 2
Solve p + q = 1

Solution
This is of the type f (p, q) = 0.
Therefore the complete solution of this partial differential equation is in the form
z= ax + by + c …….(1)
Differentiating (1) partially with respect to x and y we get
p = a, q = b.
Substituting in the given partial differential equation we have,

IMTSINSTITUTE.COM
MATHEMATICS-I 105

a 2  b 2  1, b  1  a2

The complete solution is  


z  ax  1  a 2 y  c ….(1)

Where a and c are arbitrary constants


There is no singular integral to the partial differential equation of this type. To obtain
the general solution take c = (a).

 
 z  ax  1  a 2 y   (a ) …… (2)

Differentiating partially with respect to a we get

 a 
0  x    y   ' (a)
 …….. (3)
 1 a 
2

The eliminent of a between (2) and (3) gives the general solution.

Example 2

Solve p  q  1.

Solution
The equation is of the type f (p, q) = 0.
 The complete solution is of the form z = ax + by + c ……..(1)
Differentiating (1) partially with respect to x and y we get
p = a and q = b.

Therefore the given equation becomes a  b  1.


The complete solution is

 
z  ax  1  a y  c
2

Where a and c are arbitrary constants.


This type of equation has no singular solution.
Take c = (a).

 2
 z  ax  1  a y   (a ) ……(2)

Differentiating partially with respect to a we get


1
0  x  (1  a ) y   ' (a) ………(3)
a
The eliminent of ‘a’ between (2) and (3) gives the general solution.

4.1.6 Standard Type II

Equation of the type z = px + qy + f (p, q).

IMTSINSTITUTE.COM
MATHEMATICS-I 106

This type of equation may be considered as analogous to clairaut’s form y = px + f


(p) in the ordinary differential equation.
dy
Here p
dx
Now consider the equation z  ax  by  f (a, b) …..(1)

Where a and b are arbitrary constants.


Differentiating (1) partially with respect to x and y we get
p = a …………(2) q=b ……….(3)
Therefore by eliminating the arbitrary constants from (1) (2), and (3), we get,
z  px  qy  f ( p, q ) …..(4).

Therefore (1) can be considered as the complete solution of the given partial
differential equation. This type of partial differential equation is known as extended Clairaut’s
form. The complete solution consists of a two parameter family of planes. The singular
solution if it exists is a surface having the complete solution as tangent planes.

Example 1

Solve z = px + qy – qb

Solution
This equation is a partial differential equation of Clairauts type.
Therefore the complete solution is got by replacing p by a and q by b where a and b
are arbitrary contests.
(i e) the complete solution is z = ax + by + ab ………(1).
Differentiating (1) partially with respect to a and b we get
0  xb ……(2)

0 ya ……(3)

Eliminating a and b from (1), (2) and (3) we get


z   xy  xy  xy
(i.e) z  xy  0.
This gives the singular solution of the given partial differential equation and to get
the general solution,
Put b   (a ) in (1)

z  ax   (a ) y  a (a ) …..(4)
Differentiating this partially with respect to ‘a’ we get
0  x   ' (a ) y  a ' (a )   (a ) ……. (5)
Eliminating ‘a’ from (4) and (5) we get the general solution.

IMTSINSTITUTE.COM
MATHEMATICS-I 107

Example 2
2 2
Solve z = px + qv + p q
Solution
This equation is a partial differential equation of Clairauts type.
Therefore the complete solution is

z  ax  by  a 2b 2
……..(1) where a and b arbitrary constants. Differentiating
(1) partially with respect to a and b we get

0  x  2ab 2 ……(2)

0  y  2a 2b ……(3)

x  2ab 2
 2ay 2  y 2
 
4a 4 2a 3
y2 x2
zx y
3 3
2x 2y
2 2 2
z  2 3 x 3 y 3
This is the singular solution where b = f(a). The complete solution is
z  ax  f (a ) y  a  f (a )  (2) Differentiating partially with respect to ‘a’.
2 2

0  a  f ' (a ) y  2a 2 f (a ) f ' (a )  2a f (a )  Eliminant of ‘a’ from (2) and (3) gives the
2

general solution.

Example 3

Obtain the complete solution and singular solution of z  px  qy  p 2  pq  q 2 .

Solution
This equation is of clairaut’s form. Therefore the complete solution is
z  ax  by  a 2  ab  b 2 (1) where a and b are arbitrary constant.
Differentiating (1) partially with respect to a and b we get
0  x  2 a  b ( 2)
0  y  2b  a (3)
2x-y =3a and 2y- x = 3b.
2x  y 2y  x
a ,b 
3 3
Substituting this in equation (1) we get

 2 x  y   2 x  x   2 x  y  (2 x  y )(2 y  x)  2 x  x 
2 2

z x   y      
 3   3   3  9  3 

IMTSINSTITUTE.COM
MATHEMATICS-I 108

Simplifying we get 3 z  xy  x 2  y 2
This is the singular solution,

4.1.7 Standard type III

Partial differential equation of the form F (z,p,q) = 0.


For this partial differential equation we assume z = F (x + ay) as solution.
Let u  z  ay , then z  F (u )
z z u dz
Then p  
x u x du
z z u dz
q  a
y u y du
Then the given differential equation becomes

 dz dz 
F  z , , a   0 which is an ordinary differential equation of first order.
 du du 

Procedure for solving partial differential equation of the type

F(z, p, q) = 0.
Step I : Assume u = x + ay
dz dz
Step II: Replace p and q by and a respectively in the given partial
du du
differential equation.
Step III: Solve the resulting ordinary differential equation of first order.

Example 2

2 2
Solve z= p +q for complete and singular solution.

Solution

Assume z = F (x+ay)= F (u).


dz dz
then p ,q  a
du dy
The given equation becomes
2 2
 dz   dz 
z     a2  
 du   du 

IMTSINSTITUTE.COM
MATHEMATICS-I 109

2
 dz 
  (1  a )
2
=
 du 
dz z

du 1 a2
dz du
(i.e) 
z 1 a2
u
2 z  c.
1 a2
1
 (u  b)
1 a2
 4(1  a 2 ) z  ( x  ay  b) 2 (1)
This is the complete solution.
Differentiating equation (1) partially with respect a and b we get
8az = 2(x + ay + b) y (2)
0 =2 (x + ay + b) 1 (3)
Substuting (3) in (2) we get z = 0
This is the singular solution.

Example 3

Find the complete solution of p (1+q)=qz.

Solution
Assume that z = f(x+ay)
dz adz
then p q
du' du
The given equation becomes

dz  dz  dz
1  a   a z
du  du  du
adz
(ie) 1  az
du
dz az  1

du a
adz
 du
az  1
log (az-1) =u+c.
log (az-1) =x+ay+c.

IMTSINSTITUTE.COM
MATHEMATICS-I 110

This is the complete solution.

Example 4

2 2 2
Solve p z + q = 1

Solution

Assume z=F (x + ay) = F (u). then


dz dz
p ,q  a .
du du
The given equation becomes.
2 2
 dz   dz 
z    a2    1
2

 du   du 
2

z 2  dz 
 a2    1
 du 

z 2

 a 2 dz   du.
Integration we get

 z  a  a2 logz  z  a   u  c
2
z 2 2 2 2

z  z  a  a logz  z  a   2( x  ay )  c
2 2 2 2 2

4.1.8 Standard type IV

Partial differential equation of the type

a f1 ( x, p)  f 2 ( y, q)
In this type of equation z is absent. Also the terms containing p and x can be
separated from those containing q and y
Let f1(x, p) = f2 (y, q) = k (1)
(i.e.) f1 (x, p) = k. Solving for p we get, p = F1 (x).
f2(y, q) = k. Solving for q we get q = F2 (y).
z z
Also dz  dx  dy.
x x
= pdx + qdy
= F1 (x) dx + F2 (y) dy.

IMTSINSTITUTE.COM
MATHEMATICS-I 111

Integrating  dz   F ( x)dx   F ( y)dy


1 2

z   F1 ( x)dx   F2 ( y )dy  c.
This is the complete solution

Example 1

Solve p2  q2  x  y
Solution

p2  q2  x  y .
p 2  x  q 2  y  k.
 p 2  x  k ; q 2  y  k .

p   x  k .q   y  k .
dz  pdx  qdy

   
x  k dx  yk 
Integrating we get the complete solution is
2
x  k  2  2  y  k  2  c.
3 3
z
3 3

  ( x  k ) 2  ( y  k ) 2   c.
2 3 3

3  

Example 2

p 2  q 2  z 2 ( x  y ).
Solution

Dividing by z2,
2 2
 p q
      x  y.
 z z
Take Z= log z
dz
dZ 
z
The given equation becomes
2
 Z   Z 
2

      x  y.
 x   y 

IMTSINSTITUTE.COM
MATHEMATICS-I 112

2
 Z 
2
 Z 
(i.e.)    x   y     k .
 x   y 
2
 Z 
2
 Z 
   x  k ; y     k
 x   y 
Z Z
  x  k;  yk
x x
Z Z
herefore dZ  dx  dy
x y

Z   ( x  k ) 2  ( y  k ) 2   c
2 3 3
ntegrating
3  

log z   ( x  k ) 2  ( y  k ) 2   c
2 3 3

3  

Example 3
p  q  sin x  sin y.
Solution

p  sin x  sin y  q  k .
 p  k  sin x; q  sin y  k .
dz  pdx  qdy
 (k  sin x)dx  (sin y  k )dy.
Integrating we get
z  (kx  cos x)  (ky  cos y )  c.
z  k ( x  y )  (cos x  cos y )  c.
This is the complete solution.

IMTSINSTITUTE.COM
MATHEMATICS-I 113

4.1.9 Self Assessment Question – I

1. Solve p3  q3  0
…………………………………………………………………………………………
…………………………………………………………………………………………
Ans: z  a( x  y )  c

2. Solve q2  p
…………………………………………………………………………………………
…………………………………………………………………………………………

Ans: z  a 2  ay  c
3. Solve pq = z
…………………………………………………………………………………………
…………………………………………………………………………………………

Ans: 4az  ( x  ay  b) 2
x y
4.  1
p q
…………………………………………………………………………………………
…………………………………………………………………………………………

x2 y2
Ans:  b
a 1 a
5. Solve z  p ( x  q )  qy
…………………………………………………………………………………………
…………………………………………………………………………………………
Ans: z  ax  by  ab

4.2 Lagrange’s Linear Partial Differential Equations

he partial differential Pp + Qq = R where P,Q,R are functions of x, y, z and


z z
p , q  is called Langrange’s linear differential equation.
x y
We shall now consider the method of solving such an equation.
It has already been shown that a partial differential equation of this type is obtained
by eliminating the arbitrary function from the equation F(x, y, z) =0.
In other words F (u,v)=0 is the solution of the partial differential equation Pp +
Qq=R. We are now required to find the values of u and v and have the solution F(u,v)=0.
Let us assume that u(x, y, z) = c1 and v (x, y, z)= c2
be two solutions of the given linear partial differential equation. Taking the differentials of the
above two solutions,

IMTSINSTITUTE.COM
MATHEMATICS-I 114

u u u
du  du  dy  dz  0. (1)
x y z
v v v
dv  dx  dy  dz  0. (2)
x y z
dx dy
 
u v u v u v u v
 
y z z y z x x z
dz

u v u v
 
x y y x
dx dy dz
(i.e)   (3).
P Q R
It is now easily seen that u = c1 and
v=c2 are solutions of the equation (3). Then,
 (u,v) = 0. (i.e) u =  (v) is the solution of the linear partial differential equation Pp +
Qq = R. Hence
Procedure for solving Pp +Qq = R.
dx dy dz
Step 1 : From the auxiliary equations  
P Q R
Step 2: Find two independent solutions of the auxiliary equations say
u = c1 & v = c2.
Step 3: Then the solution of the given partial differential equation is
 (u, v)=0, or u =  (v).

Example 1
2 2 2
Solve x p + y q= z
Solution
This is lagrange’s linear equation
The auxiliary equations are
dx dy dz
 
x2 y2 z 2
dx dy
Consider 
x2 y
1 1
Integrating   c.
x y
dx dz
Consider 
y2 z2

IMTSINSTITUTE.COM
MATHEMATICS-I 115

1 1
Integrating   c.
y z
1 1 1 1
The solution is    ,    0.
x y y z
Example 2

Solve ( y  z ) p  ( z  x)q  x  y.
Solution

This is a lagrange’s linear equation

The auxiliary equation are


dx dy dz
 
yz zx x y
dx  dy dy  dz dz  dx dx  dy  dz
(i.e)   
x y yz zx 2( x  y  z )
Considering first two members and integrating we get
x y
c (1)
yz
Considering first and last members and integrating we get
x  y  ( x  y  z )  c
2

 The Solution is

x y 
 , ( x  y ) 2 ( x  y  z )  0
yz 

Example 3

Solve y 2 p  x2q  x2 y 2 z 2
Solution
The auxiliary equations are
dx dy dz
2
 2  2 2 2
y x x y z
st nd
Considering 1 and 2 members,

x 2 dx  y 2 dy.

x3 y 3
Integrating we get  c
3 3

IMTSINSTITUTE.COM
MATHEMATICS-I 116

x3  y 3  c
nd rd
Considering 2 and 3 members,
dz
y 2 dy 
z2
1
3
y
Integrating  c
3 z
3
y 1
  c
3 z
 3 3 y
3
1

The solution is   x  y ,    0.
 3 z

Example 5
Solve (mz  ny ) p  (nx  lz )q  ly  mx.
Solution

The auxiliary equations are


dx dy dz
 
mz  ny nx  lz ly  mx
Using multipliers x, y, z we get each ratio
xdx  ydy  zdx

x(mz  ny )  y (nx  lz )  z (lz  mx)
xdx  ydy  zdx

0
 x 2  y 2  z   c is one solution.
Also using multipliers l, m, n, we get.
ldx  mdy  ndz
each ratio 
l (mz  ny )  m(nx  lz )  n(ly  mx)
ldx  mdy  ndz

0
 lx  my  nz  c is the 2nd solution.

The general solution  ( x 2  y 2  z 2 , lx  my  nz )  0.

IMTSINSTITUTE.COM
MATHEMATICS-I 117

4.2.1 Self Assessment Question - II

1. Solve z  p tan x  q tan y  tan z


…………………………………………………………………………………………
…………………………………………………………………………………………

 sin y sin z 
Ans: Q , 0
 sin x sin x 

2. Solve p( x  y)  q( y  x  z )  z
…………………………………………………………………………………………
…………………………………………………………………………………………

 yxz
Ans: Q  x  y  z, 0
 z2 

IMTSINSTITUTE.COM
MATHEMATICS-I 118

UNIT – V

5.0 Introduction
5.1 Objectives
5.2 Laplace Transforms of some elementary function
5.2.1 First Shifting Theorem
5.2.2 Second Shifting Theorem
5.2.3 Change of Scale property
5.2.4 Self Assessment Question - I

5.3 Inverse Laplace Transform


5.3.1 Table of Inverse Laplace Transforms
5.3.2 Inverse Laplace Transforms by first shifting theorem
5.3.2 Inverse Laplace Transforms by second shifting theorem
5.3.3 Self Assessment Questions - I

5.4 Laplace Transform of Periodic Function


5.4.1 Self Assessment Questions - II

5.5 Fourier Series


5.5.1 Fourier Coefficient
5.5.2 Self Assessment Questions - III

5.6 Half range Fourier series


5.6.1 Self Assessment Question - IV

IMTSINSTITUTE.COM
MATHEMATICS-I 119

5.0 Introduction

In this chapter, we have to deal with Laplace transform in a linear transformation. as


understanding the concept of Laplace transformation and inverse Laplace transformation. In
the Laplace Transformation with Shifting theorem deal with all features in the application of
mathematics in the field of applied sciences. Students will study for the Laplace
transformation with periodic functions.

5.1 Objectives

To develop the skill in the Laplace transformation with standard types.

To create new idea to application oriented in the all applied sciences in the
engineering systems

To understanding the way of approach for the Laplace transformation in all applied
sciences.

5.2 Laplace Transform:

Let f(t) be defined for 0< t <  .The transform of f(t) is defined by

L  f (t )   e  st f (t )dt
0
Note: In the above definition, the R.H.S. integral is a function of s only ,we can
Write L  f (t ) =F(s).

Notations : Laplace transform of f(t) is denoted by L  f (t ) ,


F(s),  (s) or f (s)

Instead of s, the parameter p can also be used .

Linearity property of the Laplace Transformation

The Laplace transformation is a linear transformation

i.e., L a1 f1 (t )  a2 f 2 (t )  a1 L  f1 (t )  a2 L  f 2 (t ) Where a1 , a2 are constants.


Piecewise (or) sectionally Continuous Function

A function f(t) is said to be piecewise (or) Sectionally continuous on a closed interval


a  t  b , if is defined on that interval and is such that the interval can be divided into a
finite number of subintervals, in each of which f(t) is continuous

Functions of exponential order


A function f(t) is said to be of exponential order if lim e  st f (t )  a finite quantity.
t 

IMTSINSTITUTE.COM
MATHEMATICS-I 120

Condition for existence of Laplace transform (sufficient condition)

If f(t) is a function which is piecewise continuous on every finite interval in the


range t  0 and is of exponential order, then the Laplace transform of f(t) exists.

This is the sufficient condition but not necessary for the existence of Laplace transform.

5.2.1 Laplace transforms of some elementary functions

Sl.No. f(t) L  f (t )
1 K,a constant k/s ,s>0
2 t n
(n  1) n!
n 1
or n 1
s s
3 1 
t s
4 e at 1
sa
5 e  at 1
sa
6 Cos at s
s  a2
2

7 Sin at a
s  a2
2

8 Sinh at a
s  a2
2

9 Cosh at s
s  a2
2

Example 1

Find L t n  ,n is a positive integer and n  -1


Solution:


L t n  =  e  st t n dt
0
 n
 x  dx
L t  =  e   x 1
n
put st  x  dt  dx
0 s s s
 
1
= n 1 
e  x x ( n 1) 1dx Since   n    e  x x n 1dx
s 0 0

(n  1) n !
=  n 1 Where n > -1
s n 1 s
Example 2

Find L e at  and L e  at 

IMTSINSTITUTE.COM
MATHEMATICS-I 121

Solution

L e at
  e  st at
e dt
0


 e  ( s  a )t  1
e
 ( s a )t
= dt     , s >a
0    s  a  0 s  a

L e  at    e  ( s  a )t e at dt 
ly 1
III , s >-a
0
sa

Example 3

Find (i) L cos at and (ii) L sin at


Solution (i) L cos at   e  st cos atdt
0

 e  st 
=  2 ( s cos at  a sin at ) 
s  a
2
0
s
= 2
s  a2


(ii) L sin at   e  st sin atdt
0

 e  st 
=  2 ( s sin at  a cos at ) 
s  a
2
0
a
= 2
s  a2
Example 4

Find L sin t cos t .

Solution L sin t cos t = L sin 2t / 2


1 1 2 1
= L sin 2t  . 2  2
2 2 s 4 s 4
Example 5

Find L sin 2 2t

L sin 2 2t 
1
Solution L 1  cos 4t
2

IMTSINSTITUTE.COM
MATHEMATICS-I 122

1 1
= L 1  L cos 4t
2 2
1 s
= 
2 s 2( s  16)
2

Example 6

Find L 7e 2t  9e 2t  5t 3 

Solution

L 7e 2t  9e 2t  5t 3  = 7 L e 2t   9 L e 2t   5 L t 3 


7 9 6 16 s  4 30
=   5. 4  2 
s2 s2 s s  4 s4

5.2.1 First shifting theorem

If L  f (t )  F ( s ), then L e at f (t )  F ( s  a )

Proof

F ( s )  L  f (t )   e  st f (t )dt.
0

F ( s  a )   e  ( s  a )t f (t )dt.
0

 e e f (t ) dt.
 st at
=
0

 F ( s  a )  L e at f (t )

5.2.2 Second Shifting theorem

 f (t  a ), t  a
If L  f (t )  F ( s ), and g (t )   then L  g (t )  e F ( s ) .
 sa

 0, t  a
Proof

By definition, we have

L  g (t )   e  st g (t )dt
0
 

 e 0dt   e f (t  a)dt
 st  st
=
0 0

IMTSINSTITUTE.COM
MATHEMATICS-I 123

e
 st
= f (t  a )dt
0

 L  g (t )   e  (u  a ) s . f (u )du Put t-a = u. then dt = du
0

e
 sa  su
=e . f (u )du
0

e
 sa  st
=e . f (t )dt by property of definite integrals
0

 L  g (t )  e  sa
F (s)

5.2.3 Change of scale Property

1
If L  f (t )  F ( s ), then L  f (at )  f (s / a)
a
Proof
by the definition, we have

L  f (at )   e  st f (at )dt
0

1 (s / a) x
a 0
 e f ( x)dx Put at = x  dt = 1/a dx

1  ( s / a )t
a 0
 e f (t )dt

1
 F (s / a) Since F ( s )   e  st f (t )dt
a 0
Note:
If L  f (t )  F ( s ), then L  f  t / a   aF (as )

Example 1

Find L t 3e 2t 
Solution

L t 3e 2t   L t 3 
ss  2

 3!  6
= 4

 s  s  s  2 (6  2)
4

Example 2

Find L e 2 t
(3cos 6t  5sin 6t ) .

Solution
s 6
We have L (3cos 6t  5sin 6t )  3.  5. 2
s  36
2
s  36

IMTSINSTITUTE.COM
MATHEMATICS-I 124

3s  30
=  F (s)
s 2  36
L e 2t (3cos 6t  5sin 6t )  F ( s  2)
3( s  2)  30 3s  24
=  2
2( s  2)  36 s  4 s  40
2

Example 3

Find L et (3sinh 2t  5cosh 2t .

Solution

2 s
We have L (3sinh 2t  5cosh 2t  3  5. 2
s 4 2
s 4
6  5s
= 2  F (s)
s 4
 L et (3sinh t  5cos 2t )  F ( s  1)
6  5( s  1) 11  5s
=  2
( s  1)  4 s  2 s  3
2

Applying the change of scale property show that


s 2  2s  4 s2  s  1
L  f (2t )  If L  f (t ) 
4( s  1) 2 ( s  2) (2 s  1) 2 ( s  1)
s2  s  1
We have L  f (t )  F ( s ) 
(2 s  1) 2 ( s  1)
1
L  f (2t )  F ( s / 2)
2
 s / 2   s / 2  1
2
1
=
2  2  s / 2   1 2  s / 2   1
   
s  2s  4
2
=
 4  s  12 ( s  2) 
 

Example 4

e t  a , t  0
Find L  g (t ) where g (t )  
 0, t  a
Solution
 f (t  a ), t  0
g (t )   where F(s) = L  f (t )
 0, t  a
Here F(s) = L e  
t 1
s 1
L  g (t )  e  sa .F ( s )
e  sa
L  g (t )  ,s 1 (Using second shifting theorem)
s 1

IMTSINSTITUTE.COM
MATHEMATICS-I 125

Example 5
3
1 ( s 1)
L e  t f (3t ) 
1 e
If L  f (t )  e s
, prove that .
s ( s  1)
Solution
1 1
L  f (t )  e s  F ( s )
Given
s
 L e f (3t )  L  f (3t )s  s 1
t

1
By change of scale property, L  f (at )  F (s / a)
a
3 3
1 3e s e s
 L  f (3t )  
3 s s
3

e ( s 1)
 L e  t f (3t ) 
s 1
Example 6

 sin t  1  sin at  1
If L   tan (1/ s ) then prove that L    tan (a / s ).
 t   t 
We know by the change of scale property
1
If L  f (t )  F ( s )thenL  f (at )  F (s / a)
a
 sin t  1
Given L   tan (1/ s )
 t 
 sin at  1 1 1 1 a
L   tan ( )  tan 1 ( )
 at  a s/a a s
 sin at  1 a
L   tan ( )
 t  s

IMTSINSTITUTE.COM
MATHEMATICS-I 126

5.2.4 Self Assessment Questions - I

1. Find he Laplace transform of sin 3 2t


……………………………………………………………………………………………

……………………………………………………………………………………………
48
Ans :
( s  4)( s 2  36)
2

e3t  e 2t
2. Find he Laplace transform of
t
……………………………………………………………………………………………

……………………………………………………………………………………………
 s2 
Ans : log  
 ( s  3) 

IMTSINSTITUTE.COM
MATHEMATICS-I 127

5.3 Inverse Laplace Transform

Definition:
Inverse Laplace Transform

If F(s) is the Laplace transform of a function f(t), L  f (t )  F ( s ), then f(t) is called


the inverse Laplace transform of the function F(s) and is written as
f (t )  L1  F ( s )

L1 is called the Inverse Transformation Operator


Linear Property

Theorem

Let F1 ( s ) and F2 ( s ) be the inverse Laplace transforms of function f1 (t ) and f 2 (t )


respectively and a, b be two constants

Then L1 aF1 ( s )  bF2 ( s )  aL1  F1 ( s )  bL1  F2 ( s )


= af1 (t )  bf 2 (t )
5.3.1 Tables of Inverse Laplace Transforms

S.No. F(s) f (t )  L1  F ( s )


1 1 1
s
2 1 t
n 1
, n is a
s n!
positive integer
3 1 tn
n 1
, n  1
s (n  1)
4 1 e at
sa
5 1 e  at
sa
6 1 sin at
s  a2
2
a
7 s Cosat
s  a2
2

8 a Sinhat
s  a2
2

9 s coshat
s  a2
2

Example 1
3 2s  1
Find the inverse transform of  2 .
s 2 s 4
2

IMTSINSTITUTE.COM
MATHEMATICS-I 128

 3 2s  1  1  1  1  s  1  1 
Solution L1  2  2   3L    2L  2  L  2 
s  2 s  4 s  2 s  4 s  4
2 t sinh 2t
= 3e  2 cosh 2t 
2
Example 2

 s 1 
1  3
Find L  2   7 
.
s  2 s  3
 s 
2

Solution
 s 1     
 3 s 1  1  1  1 
L1  2   7  = L1  2   3 L    L  7 
 s  2 s  3 s 2   s  ( 2)   s  3
2
 s 2 
7
1
t2
 cos 2t  3e3t 
7
 
2
5
t2
 cos 2t  3e3t 
5 3 1 1
. .  
2 2 2 2
8t 2 t
 cos 2t  3e3t 
15 
Example 3

 3s 2  4 s  5 
1 1  2 s  3 
Find (i) L   and (ii) L  2 
 s 
4
 s 

Solution

 3s 2  4 s  5  1  1  1  1  1  1 
L1    3L  2   4 L  3   5 L  4 
s  s  s 
4
 s 

t2 t3
= 3t  4  5.
2! 3!

5t 3
 3t  2t  2

 2s  3  1  1
(ii) L1  2   2 L1    3L1  2   2  3t
 s  s s 

IMTSINSTITUTE.COM
MATHEMATICS-I 129

5.3.2 Inverse Laplace Transform by First Shifting Theorem

If L1  F ( s )  f (t ) ,then L1  F ( s  a )  e at L1  F ( s )

Note:

L1  F ( s  a )  e  at L1  F ( s )

Example 4

 3s  2 
Find L1  2 
 s  6 s  13 

3s  2 3( s  3)  7
=
s  6 s  13 ( s  3) 2  4
2

 3s  2   3( s  3)  7 
L1  2 = L1  
 s  6 s  13   ( s  3)  4 
2

 3s  7 
= e3t L1  2 
s  4

 1  s  1  1 
 L  s 2  4   7 L  s 2  4 
3t
=e
    

 7 
=e
3t
 3cos 2t  sin 2t 
2 

Example 5

 3s  7 
Find L1  2 
 s  2s  3 

3s  7 3s  7
=
s  2 s  3 ( s  1) 2  4
2

3( s  1)  10
=
( s  1) 2  4
 3s  7  1  3( s  1)  10 
L1  2 =L  
 s  2s  3   ( s  1)  4 
2

1  ( s  1)  1  1 
= 3L    10 L  
 ( s  1)  4   ( s  1)  4 
2 2

 s  t 1  1 
= 3et L1  2   10e L  2 
s  4 s  4
= e cosh 2t  5e sinh 2t
t t

IMTSINSTITUTE.COM
MATHEMATICS-I 130

 e 2t  e 2t  t e  e
2t 2 t
 t
= 3e
t
   5e    4e  e
3t

 2   2 
Example 6
14 s  10
Find the inverse Laplace transform of
49 s 2  28s  13
 5
14  s  
14 s  10  7
=
49 s  28s  13
2
 4 13 
49  s 2  s  
 7 49 
5 2 3
s (s  ) 
2 7 2 7 7
= 2
= 2 2
7 2 9 7 2 3
s   s   
 7  49  7 7

 3 
2 t  s 
 14 s  10  2 1  7 
 L1  = e 7
L  2
 49 s  28s  13  7
2
 s2   3  
   
7 

    3 
    
2 72t  L1  s  1  7 
= e 2
L  2
7   2 3   s 2   3  
 s       
  7   7  
2 72t  3t 3t 
= e
7 cos 7  sin 7 

5.3.3 Inverse Laplace Transform by second shifting theorem

 f (t  a ), t  a
If L1  F ( s )  f (t ), thenL1 e  as F ( s )  G (t ) where G (t )  
 0, t  a
Or G(t) = f(t-a) H(t-a)
Or Equivalently if L1  F ( s )  f (t ) , then L1 e  as F ( s ) = f(t-a) H(t-a)
Where H(t-a) is Heaviside ‘s unit step function

Example 7

 e 3 s 
Find L1  4
.
 ( s  2) 
Solution
1
Let F(s) =
( s  2) 4

IMTSINSTITUTE.COM
MATHEMATICS-I 131

 1 
f (t) = L1  4
 ( s  2) 
2 t 1  1  2 t t e 2t t 3
=e L  4   e 
s  3! 6
 e 3 s
 1
L1  4
= f (t  3) H (t  3)
 ( s  2)  6
1 2(t 3)
= e (t  3)3 H (t  3)
6
Example 8

 se  s 
Find L1  2 
s  9
Solution
s 1  s 
Taking F(s) = ; f(t)= L  2   cos 3t
s 92
s  9
1  se  1  s
 s
L  2  = L e F ( s )
s  9
= cos 3(t   ) H (t   )
 cos(3  t ) H (t   )
  cos 3tH (t   )

Example 9

1  e  s 
1
Show that L  2  = sin tH (t   )  sin t
 s 1 
Solution

1
Take F (s) 
s 1 2

f (t )  L  F ( s )  sin t
1

1  e  s  1
 = L  F ( s )  L e F ( s )
1 1  s
L  2
 s 1 
= sin t  f (t   ) H (t   )
= sin t  sin(t   ) H (t   )
= sin t  sin(  t ) H (t   )
= sin t  sin tH (t   ) .

5.3.4 Self Assessment Questions - I

 10 
1. Find L1  6 
 ( S  2) 
……………………………………………………………………………………………

IMTSINSTITUTE.COM
MATHEMATICS-I 132

……………………………………………………………………………………………

2 t t5
Ans : e
12

5.4 Laplace Transform of Periodic Function

Definition

Periodic Function
A function f(t) is said to be a periodic function with period P if

f(t+P) = f(t) for all values of t.

Note:

If P is a period of the function f(t) then 2P,3P,…. Are also periods

Theorem
P
1
If f(t) is a periodic function with period P , then L  f (t )   sP 
e  st f (t )dt
1 e 0
Proof
By definition

L  f (t )   e  st f (t )dt
0
P 
 L  f (t )   e  st
f (t )dt   e  st f (t )dt ………(1)
0 P
Consider the second integral on R.H.S and put t =u+P . Then dt=du and u varies
from 0 to 
 
 e  st
f (t )dt   e  s (u  P ) f (u  P )du
P 0

 e  sP  e  su f (u )du since f(u+P)=f(u)
P

=e
 sP
L  f (t )
From (1)
P
L  f (t )   e  st f (t )dt  e  sP L  f (t )
0
P

1  e sP  L  f (t )   e st f (t )dt
0
P
1
 L  f (t )   sP 
e  st f (t )dt
1  e  0
Example 1

If f (t )  t 2 , 0  t  2 and f (t  2)  f (t ) . find L  f (t ) .

IMTSINSTITUTE.COM
MATHEMATICS-I 133

Solution

Here f(t) is a periodic function with period P=2


2
1
 L  f (t )  2 s 
e  st f (t )dt
1  e  0
2
1
2 s 
 t 2 e  st dt
1  e  0
2
1  2  e  st   e  st   e  st 
 t     2  3
1  e2 s    s   2 
2t
  s   s 0
1   e 2 s   e 2 s   e 2 s  2
 L  f (t )  4    4 2   2 3  3
1  e2 s    s   s   s  s 

2  (4 s 2  4 s  2)e 2 s
=
s 3 (1  e 2 s )
5.4.1 Self Assessment Questions - III

sin t 0  t   
1. f (t )    extended periodically with period 2 and find its laplace
0   t  2 
transform

……………………………………………………………………………………………

……………………………………………………………………………………………

5.5 Fourier Series

It has been shown by Fourier that a function f(x) which has only a finite number of
discontinuities can be expressed as a trigonometric series in a given range of x in the form
a0
f ( x)   (a1 cos x  a2 cos 2 x  a3 cos 3 x  ...  an cos nx  ...)
2
+(b1 sin x  b 2 sin 2 x  b 3 sin 3 x  ...  b n sin nx  ...)
Fourier has shown that the expansion of f(x) in the above form is possible only if it satisfies
certain conditions. These conditions called Dirichlet conditions are stated below.
Let f(x) be defined in the interval c<x<c+2 with period 2 and satisfy the following
conditions.

i) f(x) is single valued.


ii) It has a finite number of discontinuities in a period of 2.
iii) It has a finite number of maxima and minima in a given period.
c  2
iv) c
f(x) dx is convergent

IMTSINSTITUTE.COM
MATHEMATICS-I 134

If f(x) satisfies the above conditions then it is possible to express f(x) as


2
1
bn 
 
0
f ( x) sin nxdx

2
1 1
=
  2 (  x) sin nxdx
0

1 cos nx sin nx
= [(  x).( )  (1)( 2 )]02
2 n n
1 1 1 1
= [  ] 
2 n n n
 2 
1
f(x) =   2 co s nx
12 n 1 n
sin2x sin3x
= sinx+   ....
2 3
 x 2
( )
2
This representation of f(x) is called a Fourier expansion or a Fourier series. Here the
coefficients a0,an,bn are called Fourier coefficients.

5.5.1 Fourier Coefficients

Let us now to find the coefficients a0,an,bn on the assumption that the series given in
(1) Is uniformly convergent in the interval c<x<c+2 and that the term by term integration is
permissible in that interval.

Integrate (1) with respect to x between the limits c and c+2


c  2 c  2 c  2  c  2 
a0
 f ( x)dx   dx   ( an cos nx)dx   ( bn cos nx)dx
c
2 c c 1 c 1

=  a0  0  0 (Using the properties)


c  2
1
 a0 
  c
f ( x)dx

Multiply both sides of (1) by cosnx and integrate with respect to x between the limits
c and c+2
c  2 c  2 c  2  c  2 
a0
 f ( x) cos nxdx   cos nxdx   ( an cos nx) cos nxdx   ( bn cos nx) cos nxdx
c c
2 c 1 c 1
c  2
=0  
c
an cos 2 nxdx  0

= a n

c  2
1
an 
 
c
f ( x) cos nxdx

Now multiply both sides of (1) by sinnx and integrate with respect to x between the limits c
and c+2

IMTSINSTITUTE.COM
MATHEMATICS-I 135

c  2 c  2 c  2  c  2 
a0
 f ( x) sin nxdx   sin nxdx   ( an cos nx) sin nxdx   ( bn sin nx) sin nxdx
c c
2 c 1 c 1
c  2
= 00 
c
bn sin 2 nxdx

= b n
c  2
1
 bn 
 
c
f ( x) sin nxdx

Thus to summarise, the fourier expansion of a function f(x) satisfying Dirichlet


conditions is given by
1
f ( x)  (  x), 0  x  2
2
Note 1: If c = 0, then the interval of definition for f(x) is (0,2) and in this case.
2
1
a0 
 
0
f ( x)dx

2
1
an 
  0
f ( x) cos nxdx

2
1
bn 
 
0
f ( x) sin nxdx

Note 2: If c = -, then the interval of definition for f(x) is (-,) and in this interval

1
a0   f ( x)dx
 

1
 
an  f ( x) cos nxdx


1
b n   f ( x) sin nxdx
 
Value of f(x) at a point:
Note: (1) The Fourier series converges to f(x) at all points where f(x) is continuous. i.e. If x =
x0 is a point of continuity, the sum of the Fourier series at x = x0 is f(x0).
(2) If x = x0 is a point of discontinuity then the value of f(x) at x = x0 is
f ( x0  0)  f ( x0  0)
2

Example 1:
1
Obtain a Fourier expansion for the function f ( x)  (  x), 0  x  2
2
Solution:
1
f ( x)  (  x), 0  x  2
2
Let us Fourier series for the function f(x) be

IMTSINSTITUTE.COM
MATHEMATICS-I 136

a0  
f ( x)    an cos nx   bn sin nx
2 n 1 n 1
2
1
Then a0 
 
0
f ( x)dx

2
1 1
=
  2 (  x)dx
0

1 x2
= [ x  ]02
2 2
1
= [2 2  2 2 ]  0
2

a0  
 f ( x)    an cos nx   bn sin nx
2 n 1 n 1

(  x) 2  2  1
(i.e.)( )    co s nx
2 12 n 1 n 2
2
1
bn 
 
0
f ( x) sin nxdx

2
1  x 2
=
4 (
0
2
) sin nxdx

1 co s nx sin nx cos nx
= [(  x) 2 .( )  2(  x)(1)( 2 )  2( 3 )]02
4 n n n
1  2
2  2
2
= [  3  3]0
4 n n n n
 -x 
1
 f(x) =   sin nx
2 n 1 n

sin2x sin3x
= sinx+   ....
2 3

Example 2:

 x 2 2  1
If f(x) = ( ) in (0,2) show that f(x) =   co s nx
2 12 n 1 n 2
Solution:
 x 2
f(x) = ( )
2
let

IMTSINSTITUTE.COM
MATHEMATICS-I 137

2
1
a0 
 
0
f ( x)dx

2 2
1 1
=
   dx 
0
  ( x   )dx
0
2
1 x
= [[ x]0n  [   x]2 ]
 2
1 2
= [[ 2 ]  [2 2  2 2 ]  [   2 ]]
 2
 
= - +  
2 2
1  3
3 2
= [  ]
4 3 3 6
2
1
an 
 
0
f ( x) cos nxdx

2
1  x 2
=
 (
0
2
) cos nxdx

1 sin nx  cos nx sin nx


= [(  x) 2 .  2(  x)(1)( )  2(1) 2 ( 3 )]02
4 n n 2
n
1 sin nx 2(  x ) cos nx 2sin nx
= [(  x) 2   ]02
4 n n 2
n 3

1 2 2 1
= [  ] 2
4 n3 n3 n
2
1
bn 
 
0
f ( x) sin nxdx

2
1  x 2
=
4 (
0
2
) sin nxdx

1 co s nx sin nx cos nx
= [(  x) 2 .( )  2(  x)(1)( 2 )  2( 3 )]02
4 n n n
1  2
2  2
2
= [  3  3]0
4 n n n n
a0  
 f ( x)    an cos nx   bn sin nx
2 n 1 n 1

(  x) 2  2  1
(i.e.)( )    co s nx
2 12 n 1 n 2

Example 3:

Determine Fourier expansion for f(x) = - 0<x<


= x -  <x<2 and show that

1 2

r 1 (2r  1)
2

8
Solution:

IMTSINSTITUTE.COM
MATHEMATICS-I 138

2
1
a0 
 0
f ( x)dx

2 2
1 1
=
   dx 
0
  ( x   )dx
0
2
1 x
=[[ x]0n  [   x]2 ]
 2
1 2
= [[ 2 ]  [2 2  2 2 ]  [   2 ]]
 2
 
= - +  
2 2
 2
1
an  [   cos nxdx   ( x   ) cos nxdx]
 0 

1  sin nx  sin nx 2 cos nx 2


= [ ( )0  (( x   ) )  1( ) ]
 n n n2
1 cos 2nx cos nx
= [  ]
 n2 n2
1 1  (1) n
= [ ]
 n2
an = 0 when n is even
1 2
= . when n is odd
 n2
 2
1
bn  [   sin nxdx   ( x   ) sin nxdx]
 0 

1 cos nx  cos nx 2 sin nx


= {[ ( )]0  [( x   )( )  (1)( 2 )]2 }
 n n n
1   
= [ (1) n   ]
 n n n
n
(-1) 2
= 
n n
The Fourier series for f(x) is
 2  cos(2r  1) x  2  (1) n
f(x)=-    sin nx
4  r 0 (2r  1) 2 n 1 n
f(-0) = - f(+0) = 0
f(  0)  f (  0) -  0 
f(x)= = =-
2 2 2
2 1   
    
 (2r  1) 2
2 4 4

1 2
Therefore 
r 1 (2r  1)
2

8

IMTSINSTITUTE.COM
MATHEMATICS-I 139

5.5.2 Self Assessment Question – IV

1. Show that for all values of x on (-  ,  ),


x sin 2 x sin 3 x sin 4 x
 sin x     .......
2 2 2 2
……………………………………………………………………………………………

……………………………………………………………………………………………

5.6 Half Range Fourier series

If a function is defined over the range 0 to , instead of 0 to 2 or - to , it may be


expanded in a series of sine terms only or of cosine terms only. This is particularly useful in
the solution of some partial differential equations whose boundary conditions may restrict us
to a series which contains sine terms only or cosine terms only. The series produced is then
called a half range fourier series. In such cases, we may have to construct a function in the
full range 0 to 2, or - to  in such a way that the fourier expansion for this series is valid in
the half range.

Suppose we are given a function f(x) defined in the interval (0, ). Here we extend
the function in the interval (-,0) and then find the fourier coefficients in the interval (-,).
Such an extended function should converge to given function f(x) between 0 to  also. For
such an extension here we consider two types of function.

I. Suppose f(x) is defined in the interval (0, ). We define a extension of f(x) as
follows and call the new function as F(x).

Let F(x) = f(x), 0<x<


And F(x) = F(-x), -<x<
Clearly F(x) is an even function in the interval -<x< and is identified with f(x) in 0<x<.

The Fourier series for F(x) is



a0  2
F ( x)    an cos nx Where a 0   f ( x)dx
2 n 1  0

2
a n   f ( x) cos nxdx
 0
Since F(x) =f(x) in 0<x<, the function f(x) in 0<x< is given by the expansion
  
Put x = , f( )  k
2 2 2
k 4 k 1 1 1
  [ 2  2  2  .....]
2  1 3 5
1 1 1 2
 2  2  2  ..... 
1 3 5 8

II. Suppose f(x) is defined in the interval 0<x<. Then we define as extension of f(x)
as follows:

F(x) = f(x), in 0<x<


=- F(-x), in -<x<
Clearly F(x) is an odd function in the interval -<x< and is identical with f(x) in 0<x<.
The Fourier series for the odd function F(x) is

IMTSINSTITUTE.COM
MATHEMATICS-I 140

 
2
F(x)=  bn sin nx Where b n   f ( x) sin nxdx
1  0
Since F(x) =f(x), in 0<x<, the Fourier series for f(x) in 0<x< is
 
2
f(x)=  bn sin nx
 0
Where b n  f ( x) sin nxdx
1

i) A Fourier cosine series of a function f(x) in 0<x<is



a0  2
f ( x)    an cos nx Where a 0   f ( x)dx
2 1  0

2
 0
an  f ( x) cos nxdx

ii) A Fourier series of f(x) on 0<x< is given by


 
2
f ( x)   bn sin nx Where b n 
 0
f ( x) sin nxdx
1

Example 1:
4k 1
a1  . ; a2  0
 12
4k 1
a3  ( 2 ); a4  0
 3
4k 1
a1  . 2 ; a6  0 etc
 5
Solution:

We note that the given series for f(x) is a cosine series for f9x) in the half range(0, ).
Therefore let us now find the half range cosine series for f(x).

2
a0   f ( x)dx
 0

2 
2 2
=
  0
xdx 
 
(  x)dx
2
2
2 x 2 2 x2
= [ ]0  [ x  ]
 2  2 2
2  2
2  2
2 2
= .  [( 2  )  (  )]
 8  2 2 8
 3
=  
4 4

=
2

IMTSINSTITUTE.COM
MATHEMATICS-I 141


2 
2 2
an 
 
0
x cos nxdx   (  x) cos nxdx

2

2 x sin nx 1.( cos nx)  2 2 sin nx


= [  ]0  [(  x)
 n n 2
 n
-cosnx 
-(-1)( )]
n2 2


2 2 cos n 2 1  cos n
= [  ]
 n2 n2
Also, a1 = 0, a3 = 0, a5 = 0…..
2 4 2 1
a2  [ 2] ( 2 )
 2  1
2 2
a4  2  2  0
4 4
2 2 2 2 4 2 1
a6  [ 2  2 ]  [ 2 2 ]   . 2
 6 6  2 .3  3
2 2 2 2 1
a10  [ 2  2 ]   [ 2 ]
 10 10  5
 2 cos 2 x cos 6 x cos10 x
 f(x) =  ( 2    .....)
4  1 32 52
Example 2:

If f(x) = kx 0 x 
2
= k( -x) 
 x ,
2
4k sin x sin 3 x sin 5 x 1 1 1 2
Show that  f(x) = [ 2  2  2  .....] and deduce that 2  2  2  ..... 
 1 3 5 1 3 5 8
Solution:
We have to find the half range sine series for f(x)

IMTSINSTITUTE.COM
MATHEMATICS-I 142


f ( x)   bn sin nx
1

2 
2 2
bn =
 
0
kx sin nxdx   k (  x) sin nxdx

2

2k x( cos nx) 1( sin nx)  2


= [  ]0
 n n2
2k  cos nx  sin nx 
+ [(  x)( )  (1)( )]
 n n2 2

 
2k sin n 2 sin n 2
= [  ]
n n2 n2

2k 2sin n 2
= [ ]
n n2

4k sin n 2
= [ ]
n n2
4k 1
a1  . 2 ; a2  0
 1
4k 1
a3  ( 2 ); a4  0
 3
4k 1
a1  . 2 ; a 6  0 etc
 5
4k sin x sin 3 x sin 5 x
 f(x) = [  2  2  .....]
 12 3 5
  
Put x = , f( )  k
2 2 2
k 4 k 1 1 1
  [ 2  2  2  .....]
2  1 3 5
1 1 1 2
 2  2  2  ..... 
1 3 5 8
5.6.1 Self Assessment Question - IV

1. If f(x) = x 2 . 0  x   , find the cosine series for f(x)


…………………………………………………………………………………………

IMTSINSTITUTE.COM
MATHEMATICS-I 143

UNIT QUESTIONS

UNIT-I QUESTIONS

2 2 1
 
1. Find the characteristic equation of the matrix A = 2 1 1 and then find A
1
 
 7 2 3

2. Find the eigen values and eigen vectors of the following matrices

 2 0 1 2 2 0
3 2 1 2 1  2 2 0
(i)   (ii) (iii)
   
2 3  2 2 3   0 0 1 

 1 1 2 

3. Verify Cayley – Hamilton Theorem for the matrix A = 2 1 3 

 3 2 3

x  2y  2  3
3x  y  2 z  1
4. Show that the equations : are consistent and solve them
2 x  2 y  3z  2
x  y  z  1

5. Solve the equation 4 x 2  24 x 2  23 x  18  0, given that the roots are in A.P.

6. Solve x  8 x  14 x 2  18 x  15  0, given that sum of two roots is equal to the sum of


4 3

the other two.

7. Solve x 4  4 x 3  2 x 2  12 x  9  0 given it has two pairs of equal roots

8. Determine a and b so that the equation x 4  4 x 3  ax 2  4 x  b  0 has two pair of


equal roots, find the roots.

9. If  is a root of the equation , x 3  x 2  2 x  1  0 show that  2  2 is also a roots.

10. If ,  ,  ,  are the roots of x 4  7 x 3  8 x 2  5 x  10  0 , find the value of


(   2 ) (   2) (  2) (  2)
2 2 2 2

ANSWERS:

IMTSINSTITUTE.COM
MATHEMATICS-I 144

1  4 3 
 2  5  2 =0, A1 =
2  2 1
1

1 1
2. i)   1,5; x1  K1   ; X 2  K 2  
 1 1
ii)   3,3,5; x1  [3K 2  2 K1 ; k1k2 ]; X 2  K [1, 2, 1]
1
(iii) 1, 2, 2 ; K[0,0,1]; K K1[1, , 0]
2
3. ---

4. -1, 4, 4
1 9
5.  , 2,
2 2
6. 5, -1, 3, 1

7. ---
8. a = 2, b = 1, 1 2 ; 1 3
9. x  x  2 x  1  0
3 2

10. Product of the roots – 166

UNIT-II QUESTIONS

1.
th
Find the n derivative of Cos 4 x

2.
th
Find the n derivative of x 2 sin 5 x

y  2u  2u
3. If u  sin 1 s.t. 
x xy xy

 2u
4. u  x  y  z  3xuz , find 2
2 2 2

x

 x4  y 4  u u
5. if u  log   show that x  y 3
 x y  x y

Ans:

1  n   n  n 
1. yn   4.2n cos  2 x     4 cos  4 x  
8  2   2 

2. yn  5n  2

 n   
2 
  25 x sin  5 x 

 
   10nx sin 5 x  n  2   n(n  1) sin(5 x  n  2)

 2 2 2

IMTSINSTITUTE.COM
MATHEMATICS-I 145

3 proof

4. 6x

5. 3

UNIT-III QUESTIONS

e
x
1. Evaluate sin xdx


2
dx 
2. Prove that  1  tan
0
3
x

4


4

 Cos x sin
5 5
3. Evaluate : xdx
0

4. Solve : ( D 2  4) y  x 2 sin 2 x

5. Solve : ( D 2  D) y  3xe x

ANSWERS:

ex
1. I (sin x  cos x)  c
2
3. 1/60
1  x 2 cos 2 x   x 3 x 
4. y  A cos 2 x  B sin 2 x        sin 2 x
8  32   3 8 
3e x  x 2 
5. y  A  Be 
x
  x
2  2 

UNIT-IV QUESTIONS

1. Solve : z  p2  q2

2. Solve : Sin x – p = - Siny + q

3. Solve : x2 p  y 2q  z 2  0

IMTSINSTITUTE.COM
MATHEMATICS-I 146

4. Solve : p  3q  5 z  tan(q  3 x)

5. Solve z( x  y)  x2 p  y 2q

ANSWERS
1. Z=0
2. Z = K ( X +Y) – ( Cos x + Cos y) +c

1 1 1 1
3. Q  ,    0
x y y z
 1 
4. Q  y  3 x, log 5 z  tan( y  3 x)   0
 5 
1 1 x y
5. Q  , 0
 x y 2 

UNIT-V QUESTIONS
1. Find the Laplace transform of
Sinht
(i) sinh 5t (ii )
t
 1 
2. Find L1  2 
 S (S  a ) 
2

 1 S 
3. Find the Inverse Laplace Transform of log  
 S 

4. Graph the following function and find its lapalce transform

t 0t  
f (t )   ...  where f(t) = f(t + 2  )
   t   t  2 

5. Obtain the fourier series for f ( x)  x sin xin0  x  2

6. Find the sine Series for the function

f (t )  1 in 0  t  

ANSWERS:
5 1  S 1
1. (i ) ( ii) log  
S  25
2
2  S 1 
1  cos at
2.
a2
1  et
3. f (t ) 
t

IMTSINSTITUTE.COM
MATHEMATICS-I 147

1  (1  S )e  s
4.
s 2 (1  e  s

1 cos nx
5. f ( x)  1   sin x  cos x   2
2 n2 n  1

4  sin x sin 3 x sin 5 x 


6. f ( x)      .....
 1 3 5 

IMTSINSTITUTE.COM

You might also like